Sei sulla pagina 1di 94

Rourkela Workshop

PRMO
Daily Practice Problems
Sub. : Mathematics DPP :

Arithmetic Ratios
1. Tarun and Mayank run in opposite directions on a circular track, starting at diametrically opposite
points. They first meet after Tarun has run 100 meters. They next meet after Mayank has run 150
meters past their first meeting point. Each girl runs at a costant speed. What is the length of the track in
meters after dividing it by 10 ?
2. The two digits in Sunil’s age are the same as the digits in Ramesh’s age, but in reverse order. In five
years Sunil will be twice as old as Ramesh will be then. What is the difference in their current ages?
3. Driving at a constant speed, Kavita usually takes 180 minutes to drive from her house to her mother’s
1
house. One day Kavita begins the drive at her usual speed, but after driving of the way, she hits a bad
3
snowstorm and reduces her speed by 20 km per hour. This time the trip takes her a total of 276 minutes.
H
Kavita is now H kms far from her mother’s house. What is ?
5
4. At kota Summer Camp, 60% of the children play soccer, 30% of the children swim, and 40% of the
soccer players swim. To the nearest whole percent, what percent of the non-swimmers play soccer ?
5. Amit and Rahul are 20 kilometers a part. They bike toward one another with Amit travelling three times
as fast as Rahul, and the distance between them decreasing at a rate of 1 kilometers per minute. After 5
minutes, Amit stops biking because of a flat tire and waits for Rahul. After how many minutes from the
time they started to bike does Rahul reach Amit ?
6. Yesterday Ram drove 1 hour longer than Shyam at an average speed 5 miles per hour faster than
Shyam. Akash drove 2 hours longer than Shyam at an average speed 10 miles per hour faster than
x
Shyam. Ram drove 70 miles more than Shyam. If Akash drove x km more than Shyam then find
10
7. The saxena family consists of a mother, a father and some children. The average age of the members of
the family is 20, the father is 48 years old, and the average age of the mother and children is 16. How
many children are in the family ?
8. A teacher gave a test to a class in which 10% of the students are juniors and 90% are seniors. The
average score on the test was 84. The juniors all received the same score, and the average score of the
seniors was 83. What score did each of the juniors receive on the test ?
9. In the five- sided star shown, the letters A, B, C, D and E are replaced by the numbers 3, 5, 6, 7, and 9
although not necessarily in this order. The sums of the numbers at the ends of the line segments AB ,
BC , CD , DE and EA form an arithmetic sequence, although not necessarily in this order. What is
the middle term of the arithmetic sequence ?
A

C D

E B
H.O. 92, Rajeev Gandhi Nagar, Kota (Raj.) Mob. 97831-97831, 70732-22177, Ph. 0744-2423333 www.nucleuseducation.in 1

1 1
PCCP
10. A man bought two paintings and then sold them for 300 rupees each. He made a profit of 20% for the
first painting, but a loss of 20% for the second painting. State the amount of profit or loss in rupees.
11. The two digits in Jack’s age are the same as the digits in Bill’s age, but in reverse order. In five years,
Jack will be twice as old as Bill will be then. What is the difference in their current ages ?
12. Santa and Banta start their new jobs on the same day. Santa’s schedule is 3 work-days followed by 1
rest-day. Banta’s schedule is 7 work-days followed by 3 rest-days. On how many of their first 500 days
do both have rest days on the same day ?
13. Each valve A, B, and C, when open, releases water into a tank as it own constant rate. With all three
valves open, the tank fills in 1 hour, with only valves A and C open it takes 1.5 hour, and with only
valves B and C open it takes 2 hours. The number of minutes required with only valves A and B open is
14. Ram, Shyam and Hari were on a bike and were wondering how far away the nearest town was. When Ram
said, “We are at least 6 km away,” Shyam replied, “We are at most 5km away.” Hari then remarked,
“Actually the nearest town is at most 4km away.” It turned out that none of the three statements were true.
Let d be the distance in km to the nearest town. Find the sum of all possible values of d ?
15. Mohit has a collection of 23 coins, consisting of 5-cent coins, 10-cent coins, and 25-cent coins. He has
3 more 10-cent coins than 5-cent coins, and the total value of his collection is 320 cents. How many
more 25-cent coins does Mohit have than 5-cent coins ?
16. Sameer drove 96 km in 90 minutes. His average speed during the first 30 minutes was 60 kph
(Kilometer per hour), and his average speed during the second 30 minutes was 65 kph. What was his
average speed, in kph, during the last 30 minutes ?
17. Mohini rides on a flat road at 20 kilometers per hour (kph), downhill at 30 kph, and uphill at 5kph.
Honey rides on a flat road at 30 kph, downhill at 40 kph. Mohini goes from town A to town B, a
distance of 10 km all uphill, then from town B to town C, a distance of 15 km all downhill, and then
back to town A, a distance of 20 km on the flat. Honey goes the other way around using the same route.
How many mare minutes does it take Mohini to complete the 45-km ride than it takes Honey ?
18. Every week Ramesh pays for a movie ticket and a soda out of his allowance. Last week, Ramesh’s
allowance was A Rupees. The cost of his movie ticket was 20% of the difference between A and the cost of
his soda, while the cost of his soda was 5% of the difference between A and the cost of his movie ticket. To
the nearest whole percent, what fraction of A did Ramesh pay for his movie ticket and soda ?
19. Sania set off on her bicycle to visit her friend, traveling at an average speed of 17 kilometers per hour.
When she had gone half the distance to her friend’s house, a tire went flat, and she walked the rest of
the way at 5 kilometers per hour. In all it took her 44 minutes to reach her friend’s house. If Sania had
to walk for x kilometer. Find the value of 6x.
20. A rug is made with three different colors as shown. The areas of the three differently colored regions
form an arithmetic progression. The inner rectangle is one foot wide, and each of the two shaded
regions is 1 foot wide on all four sides. What is the length in feet of the inner rectangle ?

1 1

H.O. 92, Rajeev Gandhi Nagar, Kota (Raj.) Mob. 97831-97831, 70732-22177, Ph. 0744-2423333 www.nucleuseducation.in 2

2 2
PCCP
21. Two years ago Pankaj was three times as old as his cousin Naveen. Two years before that, Pankaj was
four times as old as Naveen. In how many years will the ratio of their ages be 2 : 1?
22. Disha has 12 coins, each of which is a 5-cent coin or a 10-cent coin. There are exactly 17 different
values that can be obtained as combinations of one or more of her coins. How many 10-cent coins does
Disha have?
x
23. Consider the set of all fractions , where x and y are relatively prime positive integers. How many of
y
these fractions have the property that if both numerator and denominator are increased by 1, the value
of the fraction is increased by 10% ?
24. Suppose that 4 cows give 10 gallons of milk in 2 days. At this rate, how many gallons of milk will 5
cow give in 4 days?
25. Hari drives from his home to the airport to catch a flight. He drives 35 km in the first hour, but realizes
that he will be 1 hour late if he continues at this speed. He increases his speed by 15 km per hour for the
rest of the way to the airport and arrives 30 minutes early. If the distance between airport and his home
P
is P km. Find .
7
26. Divya drove her new car on a trip for a whole number of hours, averaging 55 km per hour. At the
beginning of the trip, abc km were displayed on the odometer, where abc is a 3-digit number with a ≥ 1
and a + b + c ≤ 7. At the end of the trip, where the odometer showed cba km. What is a2 + b2 + c2 ?
27. A basketball team’s players were successful on 50% of their two-point shots and 40% of their three-
points shots, which resulted in 54 points. They attempted 50% more two-point shots than three-point
shots. How many three- point shots did they attempt ?
28. It takes yuvi 60 seconds to walk down an escalator when it is not operating and only 24 seconds to walk
down the escalator when it is operating. How many seconds does it take yuvi to ride down the operating
escalator when she just stands on it?
29. Kareena walks once around a track at exactly the same constant speed every day. The sides of the track
are straight, and the ends are semicircles. The track has width 6 meters, and it takes her 36 seconds
longer to walk around the outside edge of the track than around the inside edge. If Kareena’a speed in
π
meters per second is , find k.
k

H.O. 92, Rajeev Gandhi Nagar, Kota (Raj.) Mob. 97831-97831, 70732-22177, Ph. 0744-2423333 www.nucleuseducation.in 3

3 3
PCCP
Logarithms
1. Suppose that
log2(log3(log5(log7 N))) = 11.
How many different prime numbers are factors of N ?
(1) 1 (2) 2 (3) 3 (4) 4
2. Suppose that
log2(log2(log2 x)) = 2.
How many digits are in the base-10 representation for x ?
(1) 5 (2) 7 (3) 9 (4) 11
3. How many positive integers b have the property that logb 729 is also a positive integer ?
(1) 1 (2) 2 (3) 3 (4) 4
4. Let f (n) = log2002 n2 for all positive integers n. Define
N = f(11) + f(13) + f(14).
Which of the following relations is true ?
(1) N > 1 (2) N = 1 (3) 1< N < 2 (4) N = 2
5. For some real numbers a and b, the equation
8x3 + 4ax2 + 2bx + a = 0
has three distinct positive roots, and the sum of the base-2 logarithms of the roots is 5. What is the value
of a ?
(1) – 256 (2) – 64 (3) – 8 (4) 64
6. For any positive integer n, define
log n, if log8 n is rational,
f (n) =  8
0, otherwise.
1997
What is ∑ f (n) ?
n =1

55 58
(1) log8 2047 (2) 6 (3) (4)
3 3
7. What is the value of the expression
1 1 1 1
N= + + + ... + ?
log 2 100! log 3 100! log 4 100! log100 100!
(1) 0.01 (2) 0.1 (3) 1 (4) 2
8. What is the value of the sum
S = log10(tan 1º) + log10(tan 2º) + … + log10(tan 88º) + log10(tan 89º) ?
1 1  1 1
(1) 0 (2) log10  3 (3) log10 2 (4) log10 3
2 2  2 2
a b
9. Let a ≥ b > 1. What is the largest possible value of log a + log b ?
b a
(1) – 2 (2) 0 (3) 2 (4) 3
10. The set of all real numbers x for which log2004 (log2003(log2002(log2001 x))) is defined is {x | x > c}. If
value of c is in the form (200x)200y, then the value of xy
11. For some real numbers a and b, the equation
8x3 + 4ax2 + 2bx + a = 0
has three distinct positive roots. If the sum of the base-2 logarithms of the roots is 5, what is the value
−a
of ?
8
H.O. 92, Rajeev Gandhi Nagar, Kota (Raj.) Mob. 97831-97831, 70732-22177, Ph. 0744-2423333 www.nucleuseducation.in 4

4 4
PCCP
a
12. If log (xy3) = 1 and log(x2y) = 1, If log (xy) =
then a + b = ?
b
1 1 1 1 1
13. Evaluate + + + + .
log 2 12 5 log 3 12 5 log 4 12 5 log 5 12 5 log 6 12 5
1 1
14. The solutions to the equation log3x4 = log2x8, where x is a positive real number other than or , can
3 2
p
be written as where p and q are relatively prime positive integers. What is p + q ?
q
15. The value of log37. log59. log711. log913…. log2125. log2327 is x. Find x2.
16. Driving at a constant speed, Kavita usually takes 180 minutes to drive from her house to her mother’s
1
house. One day Kavita begins the drive at her usual speed, but after driving of the way, she hits a bad
3
snowstorm and reduces her speed by 20 km per hour. This time the trip takes her a total 276 minutes.
H
Kavita is now H kms far from her mother’s house. What is ?
5
1 1 1
17. What is the value of a for which + + =1?
log 2 a log 3 a log 4 a
1
A
18. The value of (625log5 2015 ) 4 is a four digit number A. What is ?
31
m
19. The domain of the function f(x) = log1/2(log4(log1/4( log16( log1/16 x)))) is an interval of length , where
n
m and n are relatively prime positive integers. What is m + n – 152 ?

6
20. When p = k =1
k ln k, the number ep is an integer. The largest power of 2 that is a factor of ep is 2R.
What is R ?
21. There are q values of x which satisfy log10(x – 40) + log10(60 – x) < 2. What is the value of q ?
22. Let m > 1 and n > 1 be integers Suppose that the product of the solutions for x of the equation
8(lognx)(logmx) – 7lognx – 6logmx – 2013 = 0 is the smallest possible integer. What is m + n ?
x
23. Let f(x) = 1010x, g(x) = log10   , h1(x) = g(f(x)), and hn(x) = h1(hn – 1(x)) for integers n ≥ 2. The sum of
 10 
the digits of h2011(1) is a five digit multiple of 173 and can be written as 173 × Q. What is Q ?
24. The solution of the equation 7x + 7 = 8x can be expressed in the form x = logb77. What is 21b ?
25. x = abc (a 3-digit number) satisfy the equation
log 2
x + log2x + log4(x2) + log8(x3) + log16(x4) = 40. What is ab (a 2 digit number) ?
26. There are t distinct four-tuples (a, b, c, d) of rational numbers with alog102 + blog103 + clog105 + dlog107
= 2005. What is 25 × t ?
27. Let S be the set of ordered triples (x, y, z) of real numbers for which log10(x + y) = z and log10(x2 + y2)
= z + 1. There are real numbers a and b such that for all ordered triples (x, y, z) in S we have x3 + y3 =
a. 103z + b.102z. What is the value of 4(a + b) ?
1 8
28. Find the number of positive integers x that satisfy the inequality 3 + log x < .
3 3
29. Suppose that a, b and c are real numbers greater than 1. Find the value of
1 1 1
+ + .
c a b
1 + log a 2 b   1 + log b2c   1 + log c2a  
a b c

H.O. 92, Rajeev Gandhi Nagar, Kota (Raj.) Mob. 97831-97831, 70732-22177, Ph. 0744-2423333 www.nucleuseducation.in 5

5 5
PCCP
 x2  9
30. Find the number of positive integers x, where x ≠ 9, such that log x   < 6 + log 3  
9  3  x
Where x ∈ (1, 100)
1 1 1 1
31. If a > b > 1 and + = 1229 , find the value of − .
log a b log b a log ab b log ab a
10 x+y
32. If x, y > 0, logy x + logx y = and xy = 144, then is
3 2 3
1
If b > 1, x > 0 and ( 2x ) − ( 3x )
log b 2 log b 3
33. = 0, then is
x

Sequence and Series


1. A grocer makes a display of cans in which the top row has one can and each lower row has two more
cans than the row above it. If the display contains 100 cans, how many rows are there ?
(1) 5 (2) 8 (3) 6 (4) 10
2. The second and fourth terms of a geometric sequence are 2 and 6. Which of the following is a possible
first term ?
2 3 3
(1) − 3 (2) − (3) − (4) 3
3 3
3. Figures 0, 1, 2, and 3 consists of 1, 5, 13, and 25 non-overlapping unit squares, respectively. If the
pattern were continued, how many non-overlapping unit squares would there be in Figure 100 ?

Figure 0
Figure 1

Figure 2

Figure 3
(1) 10, 401 (2) 19, 801 (3) 20, 201 (4) 39, 801
4. Let 1, 4, … and 9, 16,… be two arithmetic sequences. The set S is the union of the first 2004 terms of
each sequence. How many distinct numbers are in S ?
(1) 3722 (2) 3732 (3) 3914 (4) 3924
5. Let a1, a2, … , ak be a finite arithmetic sequence with
a4 + a7 + a10 = 17,
a4 + a5 + a6 + a7 + a8 + a9 + a10 + a11 + a12 + a13 + a14 = 77,
and ak = 13. What is k ?
(1) 16 (2) 18 (3) 20 (4) 22 (5) 24
6. A sequence of three real numbers forms an arithmetic sequence whose first term is 9. If the first term is
unchanged, 2 is added to the second term, and 20 is added to the third term, then the three resulting
numbers form a geometric sequence. What is the smallest possible value for the third term of the
geometric progression ?
(1) 1 (2) 4 (3) 36 (4) 49
H.O. 92, Rajeev Gandhi Nagar, Kota (Raj.) Mob. 97831-97831, 70732-22177, Ph. 0744-2423333 www.nucleuseducation.in 6

6 6
PCCP
7. Consider the sequence of numbers : 4, 7, 1, 8, 9 ,7, 6, ….For n > 2, the nth term of the sequence is the
units digit of the sum of the two previous terms. Let Sn denote the sum of the first n terms of this
sequence. What is the smallest value of n for which Sn > 10,000 ?
(1) 1992 (2) 1999 (3) 2001 (4) 2002
8. Suppose that the sequence {an} is defined by
a1 = 2, and an + 1 = an = 2n, when n ≥ 1.
What is a100 ?
(1) 9900 (2) 9902 (3) 9904 (4) 10100
9. The increasing sequence of positive integers a1, a2, a3, … has the property that an + 2 = an + an + 1, for all n
≥ 1. Suppose that a7 = 120. What is a8 ?
(1) 128 (2) 168 (3) 193 (4) 194
10. A sequence of three real numbers forms an arithmetic progression with a first term of 9. If 2 is added to
the second term and 20 is added to the third term, the three resulting numbers form a geometric
progression. What is the largest possible value for the third term in the geometric progression ?
11. In the sequence 2001, 2002, 2003, …., each term after the third is found by subtracting the previous
term from the sum of the two terms that precede that term. For example, the fourth term is 2001 + 2002
– 2003 = 2000. What is the 2004th term in this sequence ?
12. Consider the sequence of numbers : 4, 7, 1, 8, 9, 7, 6, …. For n > 2, the nth term of the sequence is the
units digit of the sum of the two previous terms. Let Sn denote the sum of the first n terms of this
sequence. If smallest value of n for Sn > 10,000 is abcd then find a + b + c + d:
16 108
13. Let x and y be positive real numbers. What is the smallest possible value of + + xy ?
x y
14. Find the value of the positive integer n if
1 1 1 1
+ + + ... + = 5.
4+ 5 5+ 6 6+ 7 n + n +1
15. Consider a sequence of real number {an} defined by
an
a1 = 1 and an + 1 = for n ≥ 1.
1 + na n
1
Find the value of – 2009000.
a 2005
16. James calculates the sum of the first n positive integers and finds that the sum is 5053. If he has counted
one integer twice, which one is it ?
17. What is the value of
 1 1 1 
2100 – (x + 1)(x + 2006)  + + .... +
(x + 2005)(x + 2006) 
?
 (x − 1)(x + 2) (x + 2)(x + 3)
18. Suppose an denotes the last two digits of 7n. For example, a2 = 49, a3 = 43. The value of a1 + a2 + a3
+…... + a2007 is given by x. Find sum of all the digits of x.
1 + a n −1
19. A sequence {an} is defined by a1 = 2, an = , n ≥ 2. Find the value of (1100 + 2008 a2007).
1 − a n −1
100
20. Find the minimum value of ∑ n−k
k =1
, where n ranges over all positive integers.

21. The sum of an infinite geometric series is a positive number S, and the second term in the series is 1.
What is the smallest possible value of S2 ?
22. Let a < b < c be three integers such that a, b, c is an arithmetic progression and a, c, b is a geometric
progression. What is the smallest possible value of 10c ?
H.O. 92, Rajeev Gandhi Nagar, Kota (Raj.) Mob. 97831-97831, 70732-22177, Ph. 0744-2423333 www.nucleuseducation.in 7

7 7
PCCP
23. The sequence S1, S2, S3, ……S10 has the property that every term beginning with the third is the sum of
the previous two. That is, Sn = Sn – 2 + Sn – 1 for n ≥ 3. Suppose that S9 = 110 and S7 = 42. What is S4 ?
x
24. The sequence log12162, log12x, log12y, log12z, log121250, is an arithmetic progression. What is ?
10
25. The internal angles of quadrilateral ABCD form an arithmetic progression. Triangles ABD and DCB
are similar with ∠DBA = ∠DCB and ∠ADB = ∠CBD. Moreover, the angles in each of these two
triangles also form an arithmetic progression. In degrees, the largest possible sum of two largest angles
R
of ABCD is R. What is ?
3
26. Two non-decreasing sequences of nonnegative integers have different first terms, Each sequence has
the property that each term beginning with the third is the sum of the previous two terms, and the
seventh term of each sequence is N. The smallest possible value of N is n. What is half of n ?
27. The first four terms of an arithmetic sequence are p, 9, 3p – q, and 3p + q. What is the sum of digits of
the 2010th term of the sequence ?
28. Let a + ar1 + ar12 + ar13 ….. and a + ar2 + ar22 + ar23 + ….. be two different infinite geometric series of
positive numbers with the same first term. The sum of the first series is r1, and the sum of the second
series is r2. What is 31(r1 + r2) ?
29. For each positive integer n, the mean of the first n terms of a sequence is n. What will be the answer
when 2008th term of the sequence is divide by 55?
30. The geometric series a + ar + ar2 + …. has a sum of 7, and the terms involving odd powers of r have a
sum of 3. What is 10(a + r)?
31. Find the maximum value of x − 144 + 722 − x .
99
(−1) k +1
32. If S = ∑ , find the value of 10S.
k =1 k(k + 1) ( k +1 − k )
33. Let a1, a2, a3,….. be the sequence of all positive integers that are relatively prime to 75, where a1 < a2 <
a3 < ….. (The first five terms of the sequence are : a1 = 1, a2 = 2, a3 = 4, a4 = 7, a5 = 8.)
Then the value of a2008 is defined as p103 + q102 + r10 + s. Find (p + q + r + s)
34. Given that
x + (1 + x)2 + (1 + x)3 + … + (1 + x)n = a0 + a1x + a2x2 + … + anxn,
where each ar is an integer, r = 0, 1, 2,…, n.
n(n + 1)
Find the value of n such that a0 + a2 + a3 + a4 + … + an – 2 + an – 1 = 60 – .
2
a n −1
35. Given that an + 1 = , where n = 1, 2, 3, … and a0 = a1 = 1, find the value of
1 + na n −1a n
 1 
 2 × 10 −
4
.
 a199 a 200 
36. Suppose that (un) is a sequence of real numbers satisfying un + 2 = 2un + 1 + un, and that u3 = 9 and u6 =
128. what is u5 ?
37. For each positive integer n, the mean of the first n terms of a sequence is n. What is the square root of
2008th term of the sequence to the nearest integer ?
38. A finite sequence of three-digit integers has the property that the tens and units digits of the each terms
are, respectively, the hundreds and tens digits of the first term. For example, such a sequence might
begin with terms 247, 475, and 756 and end with the term 824 . Let S be the sum of all the terms in the
sequence. What is the largest prime number that always divides S ?

H.O. 92, Rajeev Gandhi Nagar, Kota (Raj.) Mob. 97831-97831, 70732-22177, Ph. 0744-2423333 www.nucleuseducation.in 8

8 8
PCCP
39. How many non-similar triangle have angles whose degree measures are distinct positive integers in
arithmetic progression ?
40. Let a1, a2,…. be a sequence for which
a
a1 = 2 a2 = 3 and an = n −1 for each positive integer n ≥ 3.
a n −2
What is a2006?
41. The first term of a sequence is 2005. Each succeeding term is the sum of the cubes of the digits of the
T
previous terms. Find 2005
10
42. A sequence of three real numbers forms an arithmetic progression with a first term of 9. If 2 is added to
the second term and 20 is added to the third term, the three resulting numbers form a geometric
progression. What is the smallest possible value for the third term of the geometric progression ?
43. Let a1, a2, …, be a sequence with the following properties.
I. a1 = 1, and
II. a2n = n . an for any positive integer n.
Find the sum of digits of ‘n’ if a2100 = 2n.
44. In the sequence 2001, 2002, 2003,…, each term after the third is found by sub-tracting the previous
term from the sum of the two terms that precede that term. For example, the fourth term is 2001 + 2002
– 2003 = 2000. What is the 2004th term in this sequence ?
45. Let 1,4, … and 9, 16, … be two arithmetic progressions. The set S is the union of the first 2004 terms of
each sequence. Suppose there are ‘n’ distinct numbers in set ‘S’, then find the sum of the digits of ‘n’.
46. Suppose that {an} is an arithmetic sequence with a1 + a2 + …. a100 = 100 and a101 + a102 + … + a200 =
200. What is the value of 103(a2 – a1) ?
47. Let {ak} be a sequence of integers such that a1 = 1 and am + n = am + an + mn, for all positive integers m
and n. Then a12 is
48. Let a1, a2, …….ak be a finite arithemetic sequence with
a4 + a7 + a10 = 17 and a4 + a5 + a6 + …… + a12 + a13 + a14 = 77.
If ak = 13, the k =
49. Suppose x, y, z is a geometric sequence with common ratio r and x ≠ y. If x, 2y, 3z is an arithemetic
sequence, then, 27r is ?
50. The sequence
1, 2, 1, 2, 2, 1, 2, 2, 2, 1, 2, 2, 2, 2, 1, 2, 2, 2, 2, 2, 1, 2, ….
consists of 1’s separated by blocks of 2’s with n 2’s in the nth block.
The sum of digits of the sum of the first 1234 terms of this sequence is
51. Define a sequence of real numbers a1, a2, a3,….. by a1 = 1 and a 3n +1 = 99a 3n for all n ≥ 1. Then (a100)1/33 equals
52. The number of terms in an A.P. (Arithmetic Progression) is even. The sums of the odd and even
numbered terms are 24 and 30 respectively. If the last term exceeds the first by 10.5, the number of
terms in the A.P. is
53. In a geometric series of positive terms the difference between the fifth and fourth terms is 576, and the
difference between the second and first terms is 9. What is the last two digits of sum of the first five
terms of this series ?
54. Let a1, a2, ….., a2018 be a strictly increasing sequence of positive integers such that
a1 + a2 + …. + a2018 = 20182018.
What is the remainder when a13 + a 32 + ... + a 32018 is divided by 6 ?
55. A function f is defined recursively by f(1) = f(2) = 1 and f(n) = f(n – 1) – f(n – 2) + n for all integers n ≥
3. What is the value of [f(2018) – 2000] ?
H.O. 92, Rajeev Gandhi Nagar, Kota (Raj.) Mob. 97831-97831, 70732-22177, Ph. 0744-2423333 www.nucleuseducation.in 9

9 9
PCCP
56. Define a sequence recursively by F0 = 0, F1 = 1, and Fn = the remainder when Fn – 1 + Fn – 2 is divided by
3, for all n ≥ 2. Thus the sequence starts 0, 1, 1, 2, 0, 2… What is F2017 + F2018 + F2019 + F2020 + F2022 +
F2023 + F2024 ?
57. Let S(n) equal the sum of the digits of positive integer n. For example, S(1507) = 13. For a particular
positive integer n, S(n) = 1274. Find the value of [S(n + 1) – 1200] ?
58. The sum of an infinite geometric series is a positive number S, and the second term in the series is 1.
What is the smallest possible value of S ?
59. How many four digit integers abcd, with a ≠ 0, have the property that the three two-digit integers ab < bc <
cd form an increasing arithmetic sequence ? One such number is 4692, where a = 4, b = 6, c = 9, and d = 2.
60. The product (8)(888….8), where the second factor has k digits, is an integer whose digits have a sum of
1000. Find the value of (1000 – k) ?
61. Two non-decreasing sequences of nonnegative integers have different first terms. Each sequence has
the property that each term beginning with the third is the sum of the previous two terms, and the
k
seventh term of each sequences is N. If the smallest possible value of N is k, find .
2
62. Consider the set of numbers {1, 10, 102, 103, …1010}. The ratio of the largest element of the set to the
sum of the other ten elements of the set is closest to which integer ?

Exponentials and Radicals


4 4 9 9
1. The expression 4 . 9 . 4 . 9 simplifies to which of the following ?
(1) 1313 (2) 1336 (3) 3613 (4) 3636
1530
2. The expression simplifies to which of the following ?
4515
2
1
(1)   (2) 1 (3) 315 (4) 515
 
3
3. What is the value of k if 22007 – 22006 – 22005 + 22004 = k22004 ?
(1)1 (2) 2 (3) 3 (4) 4
y x
x y
4. Suppose that x > y > 0 . Which of the following is the same as ?
yy x x
x−y y−x
x x
(1) (x – y)y/x (2)   (3) 1 (4)  
y y
810 + 410
5. What is the value of ?
84 + 411
2
1
(1)   (2) 16 (3) 32 (4) 122/3
3
6. Let f(x) = x(x + 1) (x + 2)(x + 3). What is the value of
f(0) +f(–1) + f (–2) + f (–3) ?
8 10
(1) 0 (2) (3) 1 (4)
9 9
7. Which of the following values of x satisfies the expression
548/ x
25–2 = ?
526/ x.2517/ x
(1) 2 (2) 3 (3) 5 (4) 6
H.O. 92, Rajeev Gandhi Nagar, Kota (Raj.) Mob. 97831-97831, 70732-22177, Ph. 0744-2423333 www.nucleuseducation.in 10

10 10
PCCP
8. Suppose that a > 0 and b > 0. Define r to be the number that results when both the base and the exponent of
ab are tripled. Suppose now that we write r = ab.xb. Which of the following expressions represents x ?
(1) 3 (2) 3a2 (3) 27a2 (4) 2a3b
9. What is the sum of all the real numbers x that satisfy
(2x – 4)3 + (4x – 2)3 = (4x + 2x – 6)3 ?
5 7
(1) 2 (2) (3) 3 (4)
2 2
(1 – a – b)/(2 – 2b)
10. Suppose that 60a = 3 and 60b = 5. What is the value of 12 ?
(1) 3 (2) 2 (3) 5 (4) 3
11. Let ak be the coefficient of xk in the expansion of (1 + 2x)100, where 0 ≤ k ≤ 100. Find the number of
integers r : = 0 ≤ r ≤ 99 such that ar < ar + 1 .
12. Let ak, be the coefficient of xk in the expansion of
(x + 1) + (x + 1)2 + (x + 1)3 + (x + 1)4 + … + (x + 1)99.
Determine the value of [a4/a3].
13. Let a1, a2, ... be a sequence of rational numbers such that a1 = 2 and for n ≥ 1
1+ an
an + 1 = .
1 – an
Determine 30 × a2008.
m
14. If x is real, such that (2x – 4)3 + (4x – 2)3 = (4x + 2x – 6)3, if sum of all real x is , where m and n co-
n
prime, find m + n
15. For some particular value of N, when (a + b + c + d + 1)N is expanded and like terms are combined, the
resulting expression contains exactly 1001 terms that include all four variables a, b, c, and d, each to
some positive power. What is N?

H.O. 92, Rajeev Gandhi Nagar, Kota (Raj.) Mob. 97831-97831, 70732-22177, Ph. 0744-2423333 www.nucleuseducation.in 11

11 11
PCCP

Answer Key

Arithmetic Ratios
1. 35 2. 18 3. 27 4. 51 5. 65 6. 15
7. 06 8. 93 9. 12 10. 25 11. 18 12. 50
13. 72 14. 11 15. 02 16. 67 17. 65 18. 23
19. 17 20. 02 21. 04 22. 05 23. 01 24. 25
25. 30 26. 37 27. 20 28. 40 29. 3

Logarithms
Que. 1 2 3 4 5 6 7 8 9
Ans. 1 1 4 4 1 3 3 1 2
10. 12 11. 32 12. 08 13. 02 14. 31 15. 36
16. 27 17. 24 18. 65 19. 46 20. 16 21. 18
22. 12 23. 93 24. 24 25. 25 26. 25 27. 58
28. 25 29. 03 30. 80 31. 35 32. 13 33. 06

Sequence and Series


Que. 1 2 3 4 5 6 7 8 9
Ans. 4 2 3 1 2 1 2 2 4
10. 49 11. 00 12. 28 13. 36 14. 48 15. 11
16. 03 17. 95 18. 24 19. 96 20. 50 21. 16
22. 20 23. 10 24. 27 25. 80 26. 52 27. 13
28. 31 29. 73 30. 25 31. 34 32. 11 33. 20
34. 05 35. 99 36. 37. 63 38. 37 39. 59
40. 03 41. 25 42. 01 43. 18 44. 00 45. 14
46. 10 47. 78 48. 18 49. 09 50. 16 51. 99
52. 08 53. 23 54. 04 55. 17 56. 09 57. 39
58. 04 59. 17 60. 07 61. 52 62. 9

Exponentials and Radicals


Que. 1 2 3 4 5 6 7 8 9 10
Ans. 3 4 3 4 2 4 2 3 4 2
11. 67 12. 19 13. 10 14. 09
15. 14

H.O. 92, Rajeev Gandhi Nagar, Kota (Raj.) Mob. 97831-97831, 70732-22177, Ph. 0744-2423333 www.nucleuseducation.in 12

12 12
Rourkela Workshop
PRMO
Daily Practice Problems
Sub. : Mathematics DPP : 01

ALGEBRA
1. Evaluate 12 – 22 + 32 – 42 + …..– 20082 + 20092.
2. If x, y and z are positive integers such that 27x + 28y + 29z = 363, find the value of (x + y + z).
3. Given 3x2 + x = 1, find the value of 6x3 – x2 – 3x +2010.
4. Given a4 + a3 + a2 + a + 1 = 0. Find the value a2000 + a2010 + 1.
5. If (x2 – x – 1)n = a2nx2n + a2n – 1x2n – 1 + …+ a1x2 + a1x + a0, find the value of a0 + a2 + a4 + … + a2n.
6. Find a natural number n, such that 28 + 210 + 2n is a perfect square number.
7. Given that f(x) is a polynomial of degree 3, and its remainders are 2x – 5 and –3x + 4 when divided by
x2 – 1 and x2 – 4 respectively. Find the f (x).
8. Factorize x4 + y4 + (x +y)4.
9. Given that f(x) = x2 + ax + b is a polynomial with integral coefficients. If f is a common factor of
polynomials g(x) = x4 – 3x3 + 2x2 – 3x + 1 and h(x) = 3x4 – 9x3 + 2x2 + 3x – 1, find f(x).
10. When f(x) = x3 + 2x2 + 3x + 2 is divided by g(x) which is a polynomial with integer coefficients, the
quotient and remainder are both h(x). Given that h is not a constant, find g and h.
11. Suppose x – y = 1. Find the value of
x4 – xy3 – x3y – 3x2y + 3xy2 + y4.
12. If two positive integers m and n, both bigger than 1, satisfy the equation
20052 + m2 = 20042 + n2,
find the value of m + n – 200.
13. Find an integer x that satisfies the equation
x5 – 101x3 – 999x2 + 100900 = 0.
14. Let x and y be real numbers such that
x2 + y2 = 2x – 2y + 2.
What is the largest possible value of x2 + y2 – 32 ?
1
15. It is given that x = . Find the value of
2− 3
x6 – 2 3 x5 – x4 + x3 – 4x2 + 2x – 3.
2 n
16. Let f(x) = a0 +a1x + a2x + … + anx , where a1 are nonnegative integers for I = 0, 1, 2, … , n. If f(1) = 21
f (10) + 3
and f(25) = 78357, find the value of .
100
a 2 – 3a – 3
17. There are a few integer values of such that is an integer. Find the sum of all these integer
a–2
values of a .

H.O. 92, Rajeev Gandhi Nagar, Kota (Raj.) Mob. 97831-97831, 70732-22177, Ph. 0744-2423333 www.nucleuseducation.in 1

1 1
PCCP
18. Suppose that a, b, x and y are real numbers such that
ax + by = 3, ax2 + by2 = 7, ax3 + by3 = 16 and ax4 + by4 = 42.
Find the value of ax5 + by5.
19. Let a and b be two integers. Suppose x2 – x – 1 is a factor of the polynomial ax5 + bx4 + 1. Find the
value of a.
20. Suppose f is a function satisfying f(x + x–1) = x6 + x–6, for all x ≠ 0. Determine 400 – f(3).
21. Suppose x1, x2 and x3 are roots of (11 – x)3 + (13 – x)3 = (24 – 2x)3. What is the sum of x1 + x2 + x3 ?
22. Suppose that a + x2 = 2006, b + x2 = 2007 and c + x2 = 2008 and abc = 3. Find the value of
a b c 1 1 1
+ + − − − .
bc ca ab a b c
23. Find the value of
x 4 − 6x 3 − 2x 2 + 18x + 23
when x = 19 − 8 3 .
x 2 − 8x + 15
24. Consider polynomials P(x) of degree at most 3, each of whose coefficients is an element of {0, 1, 2, 3,
4, 5, 6, 7, 8, 9}. If total k polynomials satisfy P(–1) = – 9, then find k – 200 ?
25. There is a smallest positive real number a such that there exists a positive real number b such that all
the roots of the polynomial x3 – ax2 – bx – a are real. In fact, for this value of a the value of b is unique.
b3
What is this value of ?
27
26. Let P be a cubic polynomial with P(0) = k, P(1) = 2k, and P(–1) = 3k. If P(2) + P(–2) = JK. What is J ×
5?
27. Given that x and y are positive real numbers such that (x + y)2 = 2500 and xy = 500, find the exact
x 3 + y3
value of .
103
28. If | x | + x + 5y = 2 and | y | – y + x = 7, find the value of (x + y)3.
29. Let x1, x2, x3, x4 denote the four roots of the equation
x4 +kx2 + 90 x – 2009 = 0.
If x1x2 = 49, find the value of k.
1
30. Let x be a real number such that x2 – 15x + 1 = 0. Find the last two digits of x4 +
.
x4
31. Given that 169(157 – 77x)2 + 100(201 – 100x)2 = 26(77x – 157) (1000x – 2010), find the value of x.
32. If a, b, c and d are real numbers such that
b+c+d a+c+d a+b+d a+b+c
= = = = r,
a b c d
find the sum of all the possible values of r.
5 48/ x
33. Find the value of x that satisfies the equation 25–2 = .
526/ x.2517/ x
34. Let g(x) = x5 + x4 + x3 + x2 + x + 1. What is the remainder when the polynomial g(x12) is divided by the
polynomial g(x) ?
35. How many real numbers x satisfy the equation 32x + 2 – 3x + 3 – 3x + 3 = 0 ?
36. For how many integers is the number x4 – 51x2 + 50 negative ?
37. What is the sum of all the roots of the equation
5 | x | +8 = x 2 − 16 .

H.O. 92, Rajeev Gandhi Nagar, Kota (Raj.) Mob. 97831-97831, 70732-22177, Ph. 0744-2423333 www.nucleuseducation.in 2

2 2
Rourkela Workshop
PRMO
Daily Practice Problems
Sub. : Mathematics DPP : 2

Polynomials and Their Zeroes

1. Given 3x2 + x = 1, find the value of 6x3 – x2 – 3x +2010.


2. Given a4 + a3 + a2 + a + 1 = 0. Find the value a2000 + a2010 + 1.
3. If (x2 – x – 1)n = a2nx2n + a2n – 1x2n – 1 + …+ a1x2 + a1x + a0, find the value of a0 + a2 + a4 + … + a2n.
a b c
4. =
If x = = , then find the value of x.
b+c a+c a+b
5. Find a natural number n, such that 28 + 210 + 2n is a perfect square number.
6. Given that f(x) is a polynomial of degree 3, and its remainders are 2x – 5 and –3x + 4 when divided by
x2 – 1 and x2 – 4 respectively. Find the f (x).
7. Factorize x4 + y4 + (x +y)4.
8. Given that f(x) = x2 + ax + b is a polynomial with integral coefficients. If f is a common factor of
polynomials g(x) = x4 – 3x3 + 2x2 – 3x + 1 and h(x) = 3x4 – 9x3 + 2x2 + 3x – 1, find f(x).
9. For any non-negative integers m, n, p, prove that the polynomial x3m + x3n + 1 + x3p + 2 has the factor x2 +
x + 1.
10. When f(x) = x3 + 2x2 + 3x + 2 is divided by g(x) which is a polynomial with integer coefficients, the
quotient and remainder are both h(x). Given that h is not a constant, find g and h.
11. The graph of the function f is shown below. How many solutions does the equation f(f(x)) = 6 have ?
y
(–2, 6) (1, 6)
6
5
4
3
2
1
x
–7 –6 –5 –4 –3 –2–1 –1 1 2 3 4 5 6 7
–2
–3
–4
(–7, –4) –5
–6
(5, –6)
12. Suppose x – y = 1. Find the value of
x4 – xy3 – x3y – 3x2y + 3xy2 + y4.
13. If x2 + x – 1 = 0, find the value of x4 – 3x2 + 3.
14. If two positive integers m and n, both bigger than 1, satisfy the equation
20052 + m2 = 20042 + n2,
find the value of m + n – 200.

15. Find an integer x that satisfies the equation


x5 – 101x3 – 999x2 + 100900 = 0.

H.O. 92, Rajeev Gandhi Nagar, Kota (Raj.) Mob. 97831-97831, 70732-22177, Ph. 0744-2423333 www.nucleuseducation.in 1

1 1
PCCP
16. Let x and y be real numbers such that
x2 + y2 = 2x – 2y + 2.
What is the largest possible value of x2 + y2 – 32 ?
1
17. It is given that x = . Find the value of
2− 3
x6 – 2 3 x5 – x4 + x3 – 4x2 + 2x – 3 .
18. Let f(x) = a0 +a1x + a2x2 + … + anxn, where a1 are nonnegative integers for I = 0, 1, 2, … , n. If f(1) = 21
f (10) + 3
and f(25) = 78357, find the value of .
100
19. Let m ≠ n be two real numbers such that m2 = n + 2 and n2 = m + 2. Find the value of 4mn – m3 – n3.
a 2 – 3a – 3
20. There are a few integer values of such that is an integer. Find the sum of all these integer
a–2
values of a .
21. Suppose that a, b, x and y are real numbers such that
ax + by = 3, ax2 + by2 = 7, ax3 + by3 = 16 and ax4 + by4 = 42.
Find the value of ax5 + by5.
22. Let a and b be positive real numbers such that
1 1 1
– – = 0.
a b a+b
2
b a
Find the value of  +  .
a b
S
23. Let the sum of the coefficients of the polynomial is S. Find .
16
(4x2 – 4x + 3)4 (4 + 3x – 3x2)2.
24. If
1
f (x) =
x 2 + 2x + 1 + 3 x 2 – 1 + 3 x 2 – 2 x + 1
3

for all positive integers x, find the value of


f(1) + f(3) + f(5) + … + f(997) + f(999).
25. Let a and b be two integers. Suppose x2 – x – 1 is a factor of the polynomial ax5 + bx4 + 1. Find the
value of a.
26. Suppose f is a function satisfying f(x + x–1) = x6 + x–6, for all x ≠ 0. Determine 400 – f(3).
27. Suppose x1, x2 and x3 are roots of (11 – x)3 + (13 – x)3 = (24 – 2x)3. What is the sum of x1 + x2 + x3 ?
28. Suppose that a + x2 = 2006, b + x2 = 2007 and c + x2 = 2008 and abc = 3. Find the value of
a b c 1 1 1
+ + − − − .
bc ca ab a b c
29. Suppose that x – y = 1. Find the value of x4 – xy3 – x3y – 3x2y + 3xy2 + y4.
30. Find the value of
x 4 − 6x 3 − 2x 2 + 18x + 23
when x = 19 − 8 3 .
x 2 − 8x + 15
31. Let x, y, and z be three real numbers such that xy + yz + xz = 4. Find the least possible value of x2 + y2
+ z2.
32. Consider polynomials P(x) of degree at most 3, each of whose coefficients is an element of {0, 1, 2, 3,
4, 5, 6, 7, 8, 9}. If total k polynomials satisfy P(–1) = – 9, then find k – 200 ?
33. For certain real numbers a, b, and c, the polynomial g(x) = x3 + ax2 + x + 10 has three distinct roots, and
–f (1)
each root of g(x) is also a root of the polynomial f(x) = x4 + x3 + bx2 + 100x + c. What is ?
91
H.O. 92, Rajeev Gandhi Nagar, Kota (Raj.) Mob. 97831-97831, 70732-22177, Ph. 0744-2423333 www.nucleuseducation.in 2

2 2
PCCP
34. There is a smallest positive real number a such that there exists a positive real number b such that all
the roots of the polynomial x3 – ax2 – bx – a are real. In fact, for this value of a the value of b is unique.
b3
What is this value of ?
27
35. Let P be a cubic polynomial with P(0) = k, P(1) = 2k, and P(–1) = 3k. If P(2) + P(–2) = JK. What is J ×
5?
36. Consider all polynomials of a complex variable, P(z) = 4z4 + az3 + bz2 + cz + d, where a, b, c and d are
integers, 0 ≤ d ≤ c ≤ b ≤ a ≤ 4, and the polynomial has a zero z0 with |z0| = 1. What is the sum of all
values P(1) over all the polynomials with these properties ?
37. Let a > 0, and let P(x) be a polynomial with integer coefficients such that
P(1) = P(3) = P(5) = P(7) = a, and
P(2) = P(4) = P(6) = P(8) = –a.
The smallest possible value of a = 5 × Q. What is Q ?
38. Monic quadratic polynomials P(x) and Q(x) have the property that P(Q(x)) has zeroes at x = –23, –21, –
17 and –15 and Q(P(x)) has zeroes at x = –59, –57, –51, and –49. What is the last two digits of the sum
of the minimum values of P(x) and Q(x)?
39. Find the remainder when x2008 + 2008x + 2008 is divided by x2 – 3x + 2.
40. Given that x and y are positive real numbers such that (x + y)2 = 2500 and xy = 500, find the exact
x 3 + y3
value of .
103
41. Find the remainder when (x – 1)100 + (x – 2)200 is divided by x2 – 3x + 2.
42. Let p(x) be a polynomial with real coefficients such that for all real x,
2(1 + p(x)) = p(x – 1) + p(x + 1)
and p(0) = 8, p(2) = 32. Determine the value of p(4).
43. Let a, b, c, d, e be five numbers satisfying the following conditions :
a + b + c + d + e = 0, and
abc + abd + abe + acd + ace + ade + bcd + bce + bde + cde = 33.
Find the value of a3 + b3 + c3 + d3 + e3.
10x
44. Given that x and y are both negative integers satisfying the equation y = , find the maximum
10 − x
value of y + 20.
45. Given that x and y are real numbers satisfying the following equations :
x + xy + y = 2 + 3 2 and x2 + y2 = 6,
then the value of |x + y + 1| = a + b . Find the product of a and b.
46. Given that y = (x – 16) (x – 14) (x + 14) (x + 16), find the maximum value of (–0,1y).
1 1
47. Given that a + =b+ – 2 and a – b + 2 ≠ 0, find the value of ab – a + b.
a +1 b −1
48. If | x | + x + 5y = 2 and | y | – y + x = 7, find the value of (x + y)3.
49. Let x1, x2, x3, x4 denote the four roots of the equation
x4 +kx2 + 90 x – 2009 = 0.
If x1x2 = 49, find the value of k.
1
50. Let x be a real number such that x2 – 15x + 1 = 0. Find the last two digits of x4 + 4 .
x
51. If x, y and z are real numbers such that x + y + z = 9 and xy + yz + zx = 24, find the largest possible
value of z.
52. It is given that a − b = 20, where a and b are real numbers. Find the maximum possible value of
 a − 5b 
 .
 10 

H.O. 92, Rajeev Gandhi Nagar, Kota (Raj.) Mob. 97831-97831, 70732-22177, Ph. 0744-2423333 www.nucleuseducation.in 3

3 3
PCCP
2 2
53. Given that 169(157 – 77x) + 100(201 – 100x) = 26(77x – 157) (1000x – 2010), find the value of x.
(20202 − 20100)(201002 − 1002 )(20002 + 20100)
54. Evaluate .
10(20106 − 106 )
55. If a, b, c and d are real numbers such that
b+c+d a+c+d a+b+d a+b+c
= = = = r,
a b c d
find the sum of all the possible values of r.
5 48/ x
56. Find the value of x that satisfies the equation 25–2 = .
526/ x.2517/ x
57. Let the value(s) of x is such that 8xy – 12y + 2x – 3 = 0 is true for all values of y. Then find (16x).
58. If p, q and r are distinct roots x3 – x2 + x – 2 = 0, then p3 + q3 + r3 equals.
59. Let g(x) = x5 + x4 + x3 + x2 + x + 1. What is the remainder when the polynomial g(x12) is divided by the
polynomial g(x) ?
60. Find the number of pairs (m, n) of integers which satisfy the equation m3 + 6m2 +5m = 27n3 + 9n2 + 9n
+ 1.
61. How many real numbers x satisfy the equation 32x + 2 – 3x + 3 – 3x + 3 = 0 ?
62. For certain real numbers a, b, and c, the polynomial
g(x) = x3 + ax2 + x + 10
has three distinct roots, and each root of g(x) is also a root of the polynomial
f(x) = x4 + x3 + bx2 + 100x + c.
Find the value of [F(1) + 7100].
63. If y + 4 = (x – 2)2, x + 4 = (y – 2)2, and x ≠ y, what is the value of x2 + y2 ?
64. For how many integers is the number x4 – 51x2 + 50 negative ?
65. What is the sum of all the roots of the equation
5 | x | +8 = x 2 − 16 .

ANSWER KEY
1 + (–1) n 1, n = even 1
1. 2009 2. 3 3. = 4. or –1 5. n = 10
2 0, n = odd 2
5 11
6. f(x) = – x3 + 3x2 + x–8 7. 2(x2 + y2 +xy)2 8. f(x) = x2 – 3x + 1
3 3
10. g(x) = x2 + x + 1, h(x) = x + 1 11. 06 12. 01 13. 02 14. 11
15. 10 16. 06 17. 02 18. 51 19. 00 20. 08
21. 20 22. 05 23. 81 24. 05 25. 03 26. 78
27. 36 28. 01 29. 01 30. 05 31. 04 32. 20
33. 77 34. 27 35. 70 36. 92 37. 63 38. 00
39. 01 40. 50 41. 01 42. 64 43. 99 44. 15
45. 06 46. 90 47. 02 48. 27 49. 07 50. 27
51. 05 52. 50 53. 31 54. 10 55. 02 56. 03
57. 24 58. 04 59. 06 60. 00 61. 02 62. 93
63. 15 64. 12 65. 00

H.O. 92, Rajeev Gandhi Nagar, Kota (Raj.) Mob. 97831-97831, 70732-22177, Ph. 0744-2423333 www.nucleuseducation.in 4

4 4
Rourkela Workshop
PRMO
Daily Practice Problems
Sub. : Mathematics DPP :03

Quadratic Equations

1. For what values of b do the equations :


1988x2 + bx + 8891 = 0 and 8891x2 + bx + 1988 = 0 have a common root ?
2. Given that the equation in x has at least a real root, find the range of m.
(m2 – 1)x2 – 2(m + 2)x + 1 = 0
3. If the equation is x has real roots, then find the value of a and b.
x2 + 2(1 + a)x + (3a2 + 4ab + 4b2 + 2) = 0
4. Mr. Fat is going to pick three non-zero real number and Mr. Taf is going to arrange the three numbers
as the coefficients of a quadratic equation.
x2 + x+0=0
Mr. Fat wins the game if and only if the resulting equation has two distinct rational solutions. Who has
a winning strategy ?
5. a, b, c are three distinct non-zero real numbers. Prove that the following three equations ax2 + 2bx + c =
0, bx2 + 2cx + a = 0, and cx2 + 2ax + b = 0 cannot all have two equal real roots.
6. If x2 + x + 1 = 0, find the value of x1999 + x 2000.
7. For x2 + 2x + 5 to be a factor of x4 + px2 + q, find the values of p and q.
b2 + c2 + a 2
8. If a + b + c = 0, find .
b 2 − ca
9. For how many real values of a will
x2 + 2ax + 2008 = 0 has two integer roots ?
10. If x, y are positive real numbers satisfying the system of equations
x2 + y xy = 336, y2 + x xy = 112, then x + y equals
a+b
11. a, b, c are positive integers such that a2 + 2b2 – 2bc = 100 and 2ab – c2 = 100. Then is
c
12. When x is real, the greatest possible value of 10x – 100x is
13. Find integers ‘a’ and ‘b’ such that (x2 – x – 1) divides ax17 + bx16 – 1.
14. Find the real points (x, y) satisfying 3x2 +3y2 – 4xy + 10x – 10y + 10 = 0.
15. Solve for x, y and z ; if xy + x + y = 23, yz + y + z = 31, zx + z + x = 47.
16. α, β are the real roots of the equation x2 – px + q = 0. Find the number of the pairs (p, q) such that the
quadratic equation with roots α2, β2 is still x2 – px + q = 0.
17. Given that a and b are the real roots x2 – 2x – 1 = 0, find the value of 5α4 + 12β3 .
18. Given that the real numbers s, t satisfy 19s2 + 99s + 1 = 0, t2 + 99t + 19 =0, and st ≠ 1. Find the value of
st + 4s + 1
.
t
19. Given that a = 8 – b and c2 = ab – 16, prove that a = b.

H.O. 92, Rajeev Gandhi Nagar, Kota (Raj.) Mob. 97831-97831, 70732-22177, Ph. 0744-2423333 www.nucleuseducation.in 1

1 1
PCCP
2
20. Given that a, b are roots of the equation x2 – 7x + 8 = 0, where a > b . Find the value of +3b2 without
α
solving the equation.
21. Both roots of the quadratic equation x2 – 63x + k = 0 are prime numbers. The number of possible values
of k is
22. Find the sum of all possible values of a such that the following equation has real root in x :
(x – a)2 + (x2 – 3x + 2)2 = 0.
23. Let p be a real number such that the equation 2y2 – 8y = p has only one solution.
Then find the value of p + 64.
24. Let a, b and c be the lengths of the three sides of a triangle. Suppose a and b are the roots of the
equation x2 + 4(c + 2 ) = (c + 4)x,
and the largest angle of the triangle is xº. Find the value of x.
25. How many ordered pairs of integers (x, y) satisfy the equation
x2 + y2 = 2(x + y) + xy ?
26. Let n be a positive integer such that one of the roots of the quadratic equation
4x2 – (4 3 +4)x + 3 n – 24 = 0
is an integer. Find the value of n.
27. Suppose that the two roots of the equation
1 1 2
+ − =0
x 2 – 10x − 29 x 2 − 10x − 45 x 2 − 10x − 69
are α and β. Find the value of (–αβ).
28. Find total number of pairs of (x, y) where x, y belongs to integer that satisfy the equation
x + y = x2 – xy + y2.

29. Let a and b be two integers. Suppose that 7 − 4 3 is a root of the equation x2 + ax + b = 0. Find the
value of b – a.
30. Let p be an integer such that both roots of the equation
5x2 – 5px + (66p – 1) = 0
are positive integers. Find the value of p.
31. Suppose a and b are the roots of x2 + x sin α + 1 = 0 while c and d are the roots of the equation x2 + x
1 1 1 1
cos α – 1 = 0. Find the value of 2 + 2 + 2 + 2 .
a b c d
2
32. The zeroes of the function f(x) = x – ax + 2a are integers. What is the sum of all possible values of a ?
33. Let a, b, and c be three distinct one-digit numbers. The maximum value of the sum of the roots of the
abc
equation (x – a) (x – b) + (x – b) (x – c) = 0 is (where abc is a 3 digit number). What is the value
10
of a + b + c ?
34. There are exactly N distinct rational numbers k such that | k | < 200 and 5x2 + kx + 12 = 0 has at least
one integer solution for x. What is N ?
35. Let a and b be the roots of x2 +2000x + 1 = 0 and let c and d be the roots of x2 – 2008x + 1 = 0. Now P
is defined as P = (a + c) (b + c) (a – d) (b – d). Then find sum of all the digits of P.
36. Find the value of the smallest positive integer m such that the equation
x2 + 2(m + 5)x + (100m + 9) = 0
has only integer solutions.
37. Given that (m – 2) is a positive integer and it is also a factor of 3m2 – 2m + 10. Find the sum of all such
values of m.
H.O. 92, Rajeev Gandhi Nagar, Kota (Raj.) Mob. 97831-97831, 70732-22177, Ph. 0744-2423333 www.nucleuseducation.in 2

2 2
PCCP
2
38. Let a and b be the roots of the equation x – mx + 2 = 0. Suppose that a + (1/b) and b + (1/a) are the
roots of the equation x2 – px + q = 0. What is 2q ?
39. The quadratic equation x2 + mx + n = 0 has roots that are twice those of x2 + px + m = 0, and none of
n
m, n, and p is zero. What is the value of ?
p
40. What is the negative of the sum of the reciprocals of the roots of the equation
2003 1
x + 1 + =0 ?
2004 x
41. Both roots of the quadratic equation x2 – 63x + k = 0 are prime numbers. The number of possible values
of k is
42. Suppose that a and b are non-zero real numbers, and that the equation x2 + ax + b = 0 has solutions a
and b. Find the value of (a – b).
43. Let a, b, and c be real numbers such that a – 7b + 8c = 4 and 8a + 4b – c = 7. Then a2 – b2 + c2 is
44. The zeroes of the function f(x) = x2 – ax + 2a are integers. What is the sum of all possible values of a ?

ANSWER KEY
5 1
1. b =  10879 2. m ≥ – 3. a = 1, b = –
4 2
4. Mr. Fat has q winning strategy 6. –1 7. p = 6, q = 25
1
8. 2 9. 8 10. 20 11. 2 12.
4
13. a = 987, b = –1597 14. (x, y) = (–1, 1) 15. (5,3,7) or (–7, –5, –9)
16. 3 pairs are possible : (0, 0), (1, 0) and (2, 1) 17. 169
403 − 85 17
18. – 5 19. a = b = 4 20. 21. 01 22. 03
8
24. 56 24. 90 25. 06 26. 12 27. 39 28. 06
29. 05 30. 76 31. 01 32. 16 33. 12 34. 78
35. 15 36. 90 37. 51 38. 09 39. 08 40. 01
41. 01 42. 03 43. 01 44. 16

H.O. 92, Rajeev Gandhi Nagar, Kota (Raj.) Mob. 97831-97831, 70732-22177, Ph. 0744-2423333 www.nucleuseducation.in 3

3 3
Rourkela Workshop
PRMO
Daily Practice Problems
Sub. : Mathematics DPP : 4

Miscellaneous Equations

x−n x−m m
1. Solve the equation − = (where mn ≠ 0).
m n n
1
2. Given that – 2 is the solution of equation mx = 5x + (– 2)2, find the value of the expression
3
(m2 – 11m + 17)2007.
3. Solve the equation [4ax – (a + b)] (a + b) = 0, where a and b are constants.
4. If x and y can take only natural number values, find the number of (x, y) pairs satisfying the equation
2x + 5y = 100.
1 7 1 1
5. If a, b c are real numbers such that a +   = ; b +   = 4; c +   = 1, find abc.
b 3 c a
6. If a, b, c, d, satisfy the equations a + 7b + 3c + 5d = 0, 8a + 4b + 6c + 2d = –16, 2a + 6b + 4c + 8d = 16,
5a + 3b + 7c + d = – 16 then the value of (a + d) (b + c) =
7. If x + y = 5xy, y + z = 6yz, z + x = 7zx find the value of x + y + z.
8. Given that the equations in x :
  a  3x + a 1 + 4x
3  x – 2  x +   = 2x and – =0
  3  3 6
have a common solution. Find the common solution.
9. If positive numbers a, b, c satisfy abc = 1, solve the equation in x
2ax 2bx 2cx
+ + = 1.
ab + a + 1 bc + b + 1 ca + c + 1
1
10. If an + 1 = (n = 1, 2, …, 2008) and a1 = 1, find the value of a1a2 + a2a3 + a3a4 + … + a2008a2009.
1
1+
an
11. Solve the system of equations
 x − y − z =5

 y − z − x =1
z − x − y =– 15

12. Solve the system of equations
x − y + z =1
y − z + u =2

z − u + v =3
u − v + x =4

 v − x + y =5

H.O. 92, Rajeev Gandhi Nagar, Kota (Raj.) Mob. 97831-97831, 70732-22177, Ph. 0744-2423333 www.nucleuseducation.in 1

1 1
PCCP
13. Given
1 2 3
+ + = 0, (4.25)
x y z
1 6 5
− − =0 (4.26)
x y z
x y z
Find the value of + + .
y z x

14. Solve the system of equations


1 1 1
x + y + z =
2

1 1 1
 + =
y z + x 3
1 1 1
 + =
z x + y 4
15. Let f be a function with the following properties :
(i) f(1) = 1, and
(ii) f(2n) = n × f(n). for any positive integer n.
If the value of f(2100) = 249xy, then the digits x and y respectively ?
16. Find the sum of all the real numbers x that satisfy the equation
(3x – 27)2 + (5x – 625)2 = (3x + 5x – 652)2.
17. Find the largest real number x such that
x 2 + x − 1+ | x 2 − (x − 1) |
= 35x – 250.
2
18. There are n balls in a box, and the balls are numbered 1, 2, 3, …., n respectively. One of the balls is
removed from the box, and it turns out that the sum of the numbers on the remaining balls in the box is
5048. If the number on the ball removed from the box is m, find the value of m.
19. The total number of integers between 0 and 105 having the digit sum equal to 8 is n then find 5(500 –
n).
20. R nonnegative integers can be written in the form
a7.37 + a6.36 + a5.35 + a4.34 + a3.33 + a2.32 + a1.31 + a0.30,
where a1 ∈{–1, 0, 1} for ≤ I ≤ 7. What is the last two digits of R ?
21. Define a function on the positive integers recursively by f(1) = 2, f(n) = f(n – 1) + 1 if n is even. and
f(n) = f(n – 2) + 2 if n is odd and greater than 1. What is f(2017) – 2000 ?
4
3x + y  x + 3y 
22. Suppose that x and y are nonzero real numbers such that = – 2. What is the value of   ?
x – 3y  3x – y 
23. Danica drove her new car on a trip for a whole number of hour, averaging 55 miles per hour. At the
beginning of the trip, abc miles were displayed on the odometer, where abc is a 3-digit number with a ≥
1 and a + b + c ≤ 7. At the end of the trip, where the odometer showed cba miles. What is a2 + b2 + c2 ?
2 2
24. Given that x and y distinct nonzero real numbers such that x + = y + , what is 45xy – 45 ?
x x
25. Real numbers x and y satisfy the equation x2 + y2 = 10x – 6y – 34. What is (x + y)4 ?

H.O. 92, Rajeev Gandhi Nagar, Kota (Raj.) Mob. 97831-97831, 70732-22177, Ph. 0744-2423333 www.nucleuseducation.in 2

2 2
PCCP
2 2 2
26. Let a, b, and c be real numbers such that a + b + c = 2, and a – b – c = 12
p
The difference between the maximum and minimum possible values of c is with coprime p and q .
q
What is (p + q2) ?
27. Let a, b, and c be positive integers with a ≥ b ≥ c such that a2 – b2 – c2 + ab = 2011 and a2 + 3b2 + 3c2 –
3ab – 2ac – 2bc = –1997. What is a ÷ 11 ?
28. Let a, b, c, d and e be positive integers with a + b + c + d + e =2010, and let M be the largest of the
sums a + b, b + c, c + d, and d + e. The smallest possible value of M = 11 × p. What is p ?
29. Suppose that f(x + 3) = 3x2 + 7x + 4 and f(x) = ax2 + bx + c. What is (a + b + c)4 ?
30. For each positive integer n, let f(n) = n4 – 360n2 + 400. The sum of all values of f(n) that are prime
numbers is a three digit number pqr. What is p + r ?
31. Let a, b, c, d, and e be distinct integers such that (6 – a) (6 – b) (6 – c) (6 – d) (6 – e) = 45. What is a + b
+c+d+e?
32. Let m, n be integers such that 1 < m ≤ n. Define
 1  1   1   1
f(m,n) = 1 −  × 1 −  × 1 −  × .... × 1 −  .
 m   m +1  m + 2   n
S
If S = f(2, 2049) + f(3, 2049) + f(4, 2049) + … + f(2049, 2049), find the value of .
64
1 1 1 1
33. The number of positive integral solutions (a, b, c, d) satisfying + + + = 1 with the condition
a b c d
that a < b < c < d is
34. For integers a1, …., an ∈{1, 2, 3,…, 9}, we use the notation a1a 2 ...a n to denote the number 10n –1 a1 +
10n – 2 a2 + … + 10an – 1 + an. For example, when a = 2 and b = 0, ab = b2 and acbc = (ba) 2 . Find the
value of a + b + c.
35. It is known that there is only one pair of positive integers a and b such that a ≤ b and a2 + b2 + 8ab =
2010. Find the value of a + b.
 1   2   3   2010 
2010 f +f +f  + ... + f  
. Find the value of 
x 2011   2011   2011   2011 
36. Let f(x) =
x 2010
+ (1 − x) 2010 15
37. When a bucket is two-thirds full of water, the bucket and water weigh a kilograms. When the bucket is
1 1
one-half full of water the total weight is b kilograms. In terms of a and b, if a = and b = then what
3 2
is the total weight in kilograms when the bucket is full of water ?
38. Riya would take 9 hours to build a chimney alone, and Riya would take 10 hours to build it alone.
When they work together they talk a lot, and their combined output is decreased by 10 bricks per hour.
Working together, they build the chimney in 5 hours. What is the square root of the total number bricks
in the chimney ?
39. Suppose that the number a satisfies the equation 4 = a + a–1. What is the value of a 4 + a –4 to the
nearest integer ?
40. Akash and Rahul run for 30 minutes on a circular track. Akash runs clockwise at 250 m/min and uses
the inner lane with a radius of 50 meters. Rahul runs counterclockwise at 300 m/min and uses the outer
lane with a radius of 60 meters, starting on the same radius line as Akash. How many times after the
start do they pass each other ?

H.O. 92, Rajeev Gandhi Nagar, Kota (Raj.) Mob. 97831-97831, 70732-22177, Ph. 0744-2423333 www.nucleuseducation.in 3

3 3
PCCP
41. Mr. Pankaj leaves his house for work at exactly 8:00 A.M. every morning. When he averages 40 km per
hour, he arrives at his workplace three minutes late. When he averages 60 km per hour, he arrives three
minutes early. At what average speed, in km per hour, should Mr. Pankaj drive to arrive at his
workplace precisely on time ?
x –1 x – k
42. For which of the following values of k does the equation = have no solution for x ?
x – 2 x −6
1
43. If x+ x2 −1 + = 20
x − x2 −1
1
then if x2 + x4 −1 + = k, find [k]
x − x4 −1
2

4 2
44. How many ordered pairs (m, n) of positive integers are solutions to + =1?
m n
45. If x and y non-zero real numbers such that |x| + y = 3 and |x| y + x3 = 0, then find |x – y|
46. How many distinct ordered triples (x, y, z) satisfy the equations
x + 2y + 4z = 12
xy + 4yz + 2xz = 22
xyz = 6
47. How many ordered pairs of real numbers (x, y) satisfy the following system of equations ?
x+3y=3
|| x | – | y || = 1
48. What is the sum of all the solutions of x = | 2x – | 60 – 2x || ?
49. On a 50 question multiple choice math contest, students receive 4 points for a correct answer, 0 points
for an answer left blank, and –1 point for an incorrect answer. Jasvinder’s total score on the contest was
99. What is the maximum number of questions that Jasvinder could have answered correctly ?

ANSWER KEY

if a + b = 0

 n2 Then no so l ,
n

 , n≠m 
1. x =  n − m 2. – 1 3. x = if a + b ≠ 0 4. 9
no so ln , n = m  a+b
 Then
= x ,a ≠ 0
 a
 no so ln ,a = 0

13 1 1
5. 1 6. –16 7. 8. x = – is a common solution 9.
12 3 2
2008
10. 11. x = 7, y = 5, z = –3 12. x = 0, y = 6, z = 7, u = 3, v = –1
2009
23 23 23
13. –1 14. x = ,y= ,z= 15. 50 16. 07 17. 25
14 6 2
18. 02 19. 25 20. 81 21. 18 22. 16 23. 37
24. 45 25. 16 26. 25 27. 23 28. 61 29. 16
30. 10 31. 25 32. 16 33. 06 34. 18 35. 42
36. 67 37. 00 38. 30 39. 14 40. 47 41. 48
42. 05 43. 51 44. 04 45. 03 46. 06 47. 03
48. 92 49. 29
H.O. 92, Rajeev Gandhi Nagar, Kota (Raj.) Mob. 97831-97831, 70732-22177, Ph. 0744-2423333 www.nucleuseducation.in 4

4 4
Rourkela Workshop
PRMO
Daily Practice Problems
Sub. : Mathematics DPP : 05

NUMBER SYSTEM
1. If a + b + c = 66 and ab + bc + ca = 1071, where a, b and c are all prime numbers, find the value of abc.
(1) 1007 (2) 2876 (3) 1886 (4) 1765

2. The positive integers A, B, A – B and A + B are all prime numbers. The sum of these four primes is
(1) Even (2) Divisible by 3 (3) Prime (4) Divisible by 7

3. For how many positive integers a is a4 – 3a2 + 9 a prime number.


(1) 5 (2) 7 (3) 6 (4) 2

4. If n is a positive integer greater than 1, then n4 + 4 is a _____ number.


(1) Composite (2) Composite odd number
(3) Composite even number (4) Prime

5. What is the smallest positive integer x for which x2 – x + 5 is not a prime ?


(1) 5 (2) 7 (3) 6 (4) 13

6. What is the smallest composite number generated by p2 – p – 1 where p is a prime ?


(1) 155 (2) 157 (3) 156 (4) 154

7. Some prime numbers are 1 more then a square, as in 42 + 1 = 17. How many primes p ≤ 100 are of the
from p = n2 + 1?
(1) 5 (2) 7 (3) 4 (4) 23

8. How many prime numbers satisfy 300 < x < 700 ?


(1) 5 numbers (2) 3 numbers (3) 2 numbers (4) 1 numbers

51 1 1 5
9. The number of pairs of prime numbers (p,q) satisfying the condition < + < will be
100 p q 6
(1) 49 (2) 24 (3) 50 (4) 48

10. The number of positive integral values of n for which (n3 – 8n2 + 20 n – 13) is a prime number is
(1) 2 (2) 1 (3) 3 (4) 4

11. The number of positive integers ‘n’ for which 3n – 4, 4n – 5 and 5n – 3 are all primes is
(1) 1 (2) 2 (3) 3 (4) infinite

H.O. 92, Rajeev Gandhi Nagar, Kota (Raj.) Mob. 97831-97831, 70732-22177, Ph. 0744-2423333 www.nucleuseducation.in 1

1 1
PCCP
2 2
12. m, n are natural numbers. The number of pairs (m, n) for which m + n + 2mn – 2013m – 2013n –
2014 = 0 is.
(1) 2000 (2) 2013 (3) 1023 (4) 2007

13. Suppose that a, b, c are distinct numbers such that


(b – a)2 – 4(b – c)(c – a) = 0,
b−c
find the value of .
c−a

14. Suppose that p and q are prime numbers (p > q) and they are roots of the equation x2 – 99x + m = 0 for
some m. What is the value of p – q ?

15. What is the largest possible prime value of n2 – 12n + 27, where n ranges over all positive integers ?

16. What is the largest integer that is a divisor of (n + 1) (n + 3)(n + 5)(n + 7)(n + 9) for all positive even
integers n?

17. Let A be the set of positive integers that have no prime factors other than 2, 3 or 5. The infinite sum
1 1 1 1 1 1 1 1 1 1 1 1 1 1
+ + + + + + + + + + + + + + ….. of the reciprocals of the elements of A
1 2 3 4 5 6 8 9 10 12 15 16 18 20
m
can be expressed as , where m and n are relatively prime positive integers, What is m + n ?
n

18. For some positive integer n, the number 110m3 has 110 positive integer divisors, including 1 and the
D
number 110n3. The number 81n4 have D positive integer divisors, What is ?
5

19. What is the minimum number of digits to the right of the decimal point needed to express the fraction
123456789
as a decimal ?
226.54

20. How many positive two-digit integers are factors of 224 – 1 ?

ANSWER KEY

Que. 1 2 3 4 5 6 7 8 9 10 11 12
Ans. 3 3 4 1 1 1 3 3 1 3 1 2
13. 01 14. 95 15. 07 16. 15 17. 19 18. 65
19. 26 20. 12

H.O. 92, Rajeev Gandhi Nagar, Kota (Raj.) Mob. 97831-97831, 70732-22177, Ph. 0744-2423333 www.nucleuseducation.in 2

2 2
Rourkela Workshop
PRMO
Daily Practice Problems
Sub. : Mathematics DPP : 06

NUMBER SYSTEM
1. Given 1x6y7 is a five-digit number divisible by 9. The number of ordered pairs (x, y) satisfying this is
(1) 5 (2) 6 (3) 8 (4) 11
2. The number of three digit numbers that are divisible by 2 but not divisible by 4 is
(1) 200 (2) 225 (3) 250 (4) 450
3. a, b, c are the digits of a nine digit number abcabcabc. Calculate the quotient when this number is
divided by 1001001.
(1) 100a + 10c + b (2) 100c + 10b + a (3) 100a + 10b + c (4) 10a + c
n
4. Find the number of integers n for which is the square of an integer.
20 – n
(1) 1 (2) 2 (3) 3 (4) 4
5. The years of 20th century and 21th century are of 4 digits. The number of years which are divisible by
the product of the four digits of the year is
(1) 7 (2) 8 (3) 9 (4) None of these
6. The number of numbers of the form 30a0b03 that are divisible by 13, where a, b are digits, is
(1) 5 (2) 6 (3) 7 (4) 0
7. A natural number n has exactly two divisors and (n + 1) has three divisors. The number of divisors of (n
+ 2) is
(1) 2 (2) 3 (3) 4 (4) depends on the value of n
8. If n = 1010 – 1, the number of digits in n3 is
(1) 30 (2) 28 (3) 32 (4) 27
9. The last two digits of 32012, when represented in decimal notation, will be
(1) 81 (2) 01 (3) 41 (4) 21
10. Give the units digit of the number 32011.
(1) 7 (2) 8 (3) 10 (4) 11
11. When written out completely 162011 has m digits and 6252011 has n digits. Find the value of (m + n).
(1) 8000 (2) 4045 (3) 8045 (4) 7040
12. Find the units digit of (2013)2013.
(1) 2 (2) 3 (3) 5 (4) 7
100
13. Find the last two digits of 9 – 1.
14. Find the sum of digits of the largest positive integer n such that n! ends with exactly 100 zeros.
2005
15. Find the last two digits (in order) of 73 .

H.O. 92, Rajeev Gandhi Nagar, Kota (Raj.) Mob. 97831-97831, 70732-22177, Ph. 0744-2423333 www.nucleuseducation.in 1

1 1
PCCP
16. Find the smallest two-digit number N such that the sum of digits of 10N – N is divisible by 170.
17. Find the number of positive integers n such that
n + 2n2 + 3n3 + … + 2005n2005
is divisible by n – 1.

18. Let n be the number


(999 999 999...999) 2 − (666 666 666...666) 2
 
2006 6's 2006 6's

Find the reminder when n is divided by 11.


19. What is the sum of all the digits of the largest positive integer n for which n3 + 2006 is divisible by n +
26 ?
20. Find the smallest positive integer n such that n(n + 1) (n + 2) is divisible by 247.
21. Let N = 12345678910111213…4344 be the 79 digit number obtained that is formed by writing the
integers from 1 to 44 in order, one after the other. The remainder when N is divided by 45 is r. What is
r2 ?
22. Let k = 20082 + 22008. What is the square of the units digit of k2 + 2k ?
23. Anna randomly picked five integers from the following list
53, 62, 66, 68, 71, 82, 89
and discover that the average value of the five integers she picked is still an integer. If two of the
integers she picked were 62 and 89, find the mean of the remaining three integers.
24. Total number of 0’s at the end of the value of the product 1 × 2 × 3 × 4 × … × 2008 is N, then find the

value of n + 125
25. Find the remainder when (1! × 1) + (2! × 2) + (3! × 3) + … + (286! × 286) is divided by 24 × 53.
26. What is the remainder when 30 + 31 + 32 + … + 32009 is divided by 8 ?
27. Let n denote the smallest positive integer that is divisible by both 4 and 9, and whose base-10
representation consists of only 4’s and 9’s, with at least one of each. What will be the sum of the last
four digits of n ?
28. How many distinct four-digit numbers are divisible by 3 and have 23 as their last two digits ?
29. Suppose that n is the product of three consecutive integers and that n is divisible by 7. Which of the
following is not necessarily a divisor of n ?
30. An n-digit positive integer is cute if its n digits are an arrangement of the set {1, 2, …, n} and its first k
digits form an integer that is divisible by k, for k = 1, 2, …, n. For example, 321 is a cute 3-digit integer
because 1 divides 3, 2 divides 32, and 3 divids321. How many cute 6- digit integers are there ?
31. Consider the non-decreasing sequence of positive integers
1,2,2,3,3,3,4,4,4,4,5,5,5,5,5,…..
in which the nth positive integer appears n times. The remainder when the 2019th term is divided by 5 is
32. Mary chose an even 4-digit number n. She wrote down all the divisors of n in increasing order from left
n
to right : 1, 2, …, , n. At some moment Mary wrote 323 as a divisor of n. If the smallest possible
2
N
value of the next divisor written to the right of 323 is N. Find the value of .
10

H.O. 92, Rajeev Gandhi Nagar, Kota (Raj.) Mob. 97831-97831, 70732-22177, Ph. 0744-2423333 www.nucleuseducation.in 2

2 2
PCCP
33. Let N = 123456789101112… 4344 be the 79-digit number obtained that is formed by writing the
integers from 1 to 44 in order, one after the other. What is the remainder when N is divided by 45 ?
34. For some positive integer n, the number 110n3 has 110 positive integer divisors, including 1 and the
number 110n3. How many positive integer divisors does the number 9n2 have ?
35. What is the last two digits of 20152016 – 2017 ?
36. The largest divisor of 2, 014, 000, 000 is itself. If is fifth largest divisor is X. Find sum of all the digits
of X.
37. How many three-digit numbers are not divisible by 5, have digits that sum to less than 20, and have the
first digit equal to the third digit ?
38. How many even integers are there between 200 and 700 whose digits are all different and come from
the set 1, 2, 5, 7, 8, 9 ?

ANSWER KEY
DPP – 6
Que. 1 2 3 4 5 6 7 8 9 10 11 12
Ans. 4 2 3 3 4 3 2 1 3 1 3 2
13. 00 14. 13 15. 43 16. 20 17. 16 18. 00
19. 19 20. 37 21. 81 22. 36 23. 73 24. 25
25. 08 26. 04 27. 21 28. 30 29. 28 30. 02
31. 04 32. 34 33. 09 34. 63 35. 08 36. 20
37. 60 38. 12

H.O. 92, Rajeev Gandhi Nagar, Kota (Raj.) Mob. 97831-97831, 70732-22177, Ph. 0744-2423333 www.nucleuseducation.in 3

3 3
Rourkela Workshop
PRMO
Daily Practice Problems
Sub. : Mathematics DPP : 07

NUMBER SYSTEM

1. The sum of the digits of the number 10n – 1 is 3798. The value of n is
(1) 431 (2) 673 (3) 422 (4) 501
2. a, b, c, d are natural numbers such that a = bc, b = cd, c = da and d = ab. The (a + b) (b + c) (c + d) (d +
a) is equal to
(1) (a + b + c + d)2 (2) (a + b)2 + (c + d)2 (3) (a + d)2 + (b + c)2 (4) (a + c)2 + (b + d)2
3. A three digit number with digits A, B, C in that order is divisible by 9. A is an odd digit and C is an
even digit. B and C are non zero. The number of such three digit numbers is
(1) 4 (2) 8 (3) 16 (4) 20
4. Nine numbers are written in ascending order. The middle number is also the average of the nine
numbers. The average of the 5 larger numbers is 68 and the average of the 5 smaller numbers is 44. The
sum of all the numbers is
(1) 540 (2) 450 (3) 504 (4) 501
5. If the average of 20 different positive integers is 20 then the greatest possible number among these 20
numbers can be
(1) 210 (2) 200 (3) 190 (4) 180
a b
6. If a, b are natural numbers such that a + b = 2008, then (– 1) + (–1) is
(1) 1 (2) –1 (3) 2 (4) 2 or –2
12 4
7. The number 16 is obtained from the number 8 by raising the smaller number to the power n. Then n
is
16 8
(1) 3 (2) 4 (3) (4)
3 3
8. The remainder when the number
(2 × 3 × 4 × 2012 × 2013 × 2014 × 2015) – 2008 is divided by 2015 is
(1) 1 (2) 2008 (3) 7 (4) 2007
2004
1
9. The last digit in the finite decimal representation of the number   is
5
(1) 8 (2) 6 (3) 4 (4) 2
10. A natural number n has exactly two divisors and (n + 1) has three divisors. The number of divisors of (n
+ 2) is
(1) 2 (2) 3 (3) 4 (4) depends on the value of n
11. The sum of two natural number is 100. Which of the following can be the greatest product of the
numbers.
(1) 1000 (2) 4000 (3) 2500 (4) 900

H.O. 92, Rajeev Gandhi Nagar, Kota (Raj.) Mob. 97831-97831, 70732-22177, Ph. 0744-2423333 www.nucleuseducation.in 1

1 1
PCCP
12. A computer is printing a list of the seventh powers of all natural numbers, that is sequence 17, 27, 27,
……. The number of terms (or numbers) between 521 and 428 are
(1) 12 (2) 130 (3) 14 (4) 150
13. For a positive integer k, we write
(1 + x) (1 + 2x) (1 + 3x) … (1 + kx) = a0 + a1x + a2x2 + … + akxk,
where a0, a1, …, ak are the coefficients of the polynomial. Find the sum of all the digits of smallest
possible value of k if a0 + a1 + a2 + … + ak – 1 is divisible by 2005.
14. The smallest three-digit number n such that if the three digits are a, b and c, then
n = a + b + c + ab + bc + ac + abc.
n +1
Evaluate
5
15. What is the largest positive integer n satisfying n200< 5300 ?
16. N pieces of candy are made and packed into boxes, with each box containing 45 pieces. If N is a non-
zero perfect cube and 45 is one of its factors, what is the least possible number of boxes that can be
packed ?
17. For any non-empty finite set A of real numbers, let s(A) be the sum of the elements in A. There are
exactly 61 3-element subsets A of {1, … , 23} with s(A) = 36. The total number of 3-element subsets of
n + 45
{1, … , 23} with s(A) < 36 is given by N. Then evaluate
25
18. Let the sum of the digits of the given product is written as A8B6C. Find A + B + C
(111111111...111) × 2007
  
2007 1's

19. When 2007 bars of soap are packed into N boxes of equal size, where N is an integer strictly between
200 and 300, there are extra 5 bars remaining. Find last two digit of N.
20. Let the number of distinct integers among the numbers is N. Find the sum of all digits of N.
 12   22   2007 2 
  
,  , … ,  .
 2007   2007   2007 
21. For each integer n ≥ 4, let an denote the base-n number 0.133n . The product a4a5 ……… a99 can be
m
expressed as . where m and n are positive integers and n is as small as possible. What is the value of
n!
sum of the digits of m ?
22. The square is a multiplicative magic square. That is, the product of the numbers in each row, column,
and diagonal is the same. If all the entries are positive integers, what is the sum of the possible values of
g?

50 b c
d e f
g h 2

n
23. For how many integers n is the square of an integer ?
20 − n
24. How many four-digit numbers N have the property that the three-digit number obtained by removing
the leftmost digit is one ninth of N ?
25. If a, b, and c are positive real numbers such that a(b + c) = 152, b(c + a) = 162, and c(a + b) = 170, then
abc
is
12
H.O. 92, Rajeev Gandhi Nagar, Kota (Raj.) Mob. 97831-97831, 70732-22177, Ph. 0744-2423333 www.nucleuseducation.in 2

2 2
PCCP
26. If x, y, and z are positive numbers satisfying
x + 1/y = 4, y + 1/z = 1, and z + 1/x = 7/3,
then find xyz ?
n
2
27. There are x integer values of n (not necessarily positive) so that value of 4000.   is an integer.
5
2
What is x ?
28. Let S be a set of 6 integers taken from {1, 2, …. 12} with the property that if a and b are elements of S
with a < b, then b is not a multiple of a. What is the square of the least possible value of an element in
S?
29. R nonnegative integers can be written in the form
a7.37 + a6.36 + a5.35 + a4.34 + a3.33 + a2.32 + a1.31 + a0.30,
where a1∈{–1, 0, 1} for 0 ≤ | ≤ 7. What is the least two digits of R ?
30. How many odd positive 3-digit integers are divisible by 3 but do not contain the digit 3 ?
5 p 4
31. Let p and q be positive integers such that < < and q is as small as possible. What is (q – p)2 ?
9 q 7
32. A function f is defined recursively by f(1) = f(2) = 1 and f(n) = f(n – 1) – f(n – 2) + n for all integers n ≥
3. What is the sum of the digits of f(2018)?
33. Sunita has 30 thin rods, one each of every integer length from 1 cm through 30 cm. She places the rods
with lengths 3 cm, 7 cm, and 15 cm on a table. She then wants to choose a fourth rod that she scan pot
with these three to form a quadrilateral with positive area. How many of the remaining rods can she
choose as the fourth rod ?
34. A five-digit palindrome is a positive integer with respective digits abcba, where a is non-zero. Let S be
the sum of all five-digit palindromes. What is the sum of the digits of S ?
35. The products (8) (888….8), where the second factor has k digits. is an integer whose digits have a sum
of 1000. What is the sum of digits of k?
n
36. There are total P positive integers value of n, so that is also a positive integer. What is P2 ?
30 − n
1
37. Let S be the set of positive integers n for which has the repeating decimal representation 0.ab =
n
0.ababab…., with a and b different digits. The sum of the elements of S id a three digit number abc.
abc
What is the value of ?
11
38. Let a, b, and c be digit with a ≠ 0. The three-digit integer abc lies one third of the way from the square
of a positive integer to the square of the next larger integer. The integer acb lies two thirds of the way
between the same two squares. What is a + b + c ?
a 14b
39. There are p pairs of positive integers (a, b) such that gcd(a, b) = 1 and + is an integer. What is 7
b 9a
×p?
40. Let a, b, c, d, e, f, g and h be distinct elements in the set {–7, –5, –3, –2, 2, 4, 6, 13}. What is the
minimum possible value of (a + b + c + d)2 + (e + f + g + h)2
41. There are 88 numbers a1, a2, a3, …, a88 and each of them is either equals to –3 or –1. Given that
a12 + a 22 + ... + a 88
2
= 280, and a14 + a 42 + ... + a 88
4
= 103a + b. Find the product of a and b
42. The difference between the highest common factor and the lowest common multiple of x and 18 is 120.
Find the value of x.
43. Given that n is a ten-digit number in the form 2007x2008y where x and y can be any of the digits 0, 1,
2 …, 9. How many such numbers n are there that are divisible by 33 ?
H.O. 92, Rajeev Gandhi Nagar, Kota (Raj.) Mob. 97831-97831, 70732-22177, Ph. 0744-2423333 www.nucleuseducation.in 3

3 3
PCCP
n2 − 9
44. How many positive integers n, where 10 ≤ n ≤ 100, are there such that is a fraction in its lowest
n2 − 7
terms ?
45. Let n be a positive integer such that n2 + 19n + 48 is a perfect square. Find the value of n.
46. Find the smallest integer n such that n ( 101 – 10) > 1.
47. Find the smallest positive integer N such that 2n> n2 for every integer n in {N, N + 1, N + 2, N +3, N +
4}.
48. The highest natural number which are divisible by 30 and have exactly 30 different positive divisors
can be written as A1B5C then find A + B + C.
49. Let p and q represent two consecutive prime numbers. For some fixed integer n, the set {n – 1, 3n – 19,
38 – 5n, 7n – 45} represents {p, 2p, q, 2q}, but not necessarily in that order. Find the value of n.
50. If a, b and c are positive real numbers such that
ab + a + b = bc + b + c = ca + c + a = 35,
then the value of (a + 1) (b + 1) (c + 1) = xy. Find the product of x and y.
51. Find the largest possible value of n such that there exist n consecutive positive integers whose sum is
equal to 2010.
8n 3 – 96n 2 + 360n − 400
52. Let S be the set of all integers n such that is an integer. Find the value of
2n − 7
∑| n | .
n∈S

53. Find the sum of all positive integers p such that the expression (x – p) (x – 13) + 4 can be expressed in
the form (x + q) (x + r) for distinct integers q and r.
1 1 1
54. Let Pk = 1 + − 2 − 3 , where k is a positive integer. Find the sum of all the digits of least positive
k k k
integer n such that the product P2P3… Pn exceeds 2010.
55. What is the sum of the digits of the square of 111, 111, 111 ?
56. Let a, b, c and d be real numbers with | a – b | = 2, | b – c | = 3 and | c – d | = 4. What is the sum of all
possible values of | a – d | ?
57. Let k = 20082 + 22008. What is the units digit of k2 + 2k ?
58. The numbers from 1 to 8 are placed at the vertices of a cube in such a manner that the sum of the four
numbers on each face is the same. What is this common sum ?
59. The saxena family consists of a mother, a father and some children. The average age of the members of
the family is 20, the father is 48 years old, and the average age of the mother and children is 16. How
many children are in the family ?
60. How many ordered pairs (m, n) of positive integers, with m > n, have the property that their squares
differ by 96 ?
61. For each positive integer n, let S(n) denote the sum of the digits of n. For how many values of n is n +
S(n) + S(S(n)) = 2007 ?
62. How many pairs of positive integers (a, b) are there such that gcd(a, b) = 1 and
a 14b
+
b 9b
is an integer ?
63. If N is the number of four-digit positive integers have at least one digit that is a 2 or a 3 then find the
sum of the digits present in N ?
64. How many three-digit numbers satisfy the property that the middle digit is the average of the first and
the last digits ?
65. How many positive integer cubes divide 3! . 5! . 7! ?
H.O. 92, Rajeev Gandhi Nagar, Kota (Raj.) Mob. 97831-97831, 70732-22177, Ph. 0744-2423333 www.nucleuseducation.in 4

4 4
PCCP
66. The sum of the digits of a two-digit number is subtracted from the number. The units of the result is 6.
How many two-digit numbers have this property ?
67. For how many positive integers n does 1 + 2 + … + n evenly divide from 6 n ?
68. For each positive integer m > 1, let P(m) denote the greatest prime factor of m. For how many positive
integers n is it true that both P(n) = n and P(n + 8) = n + 48 ?
 a.b.c.d 
69. Suppose that 4a = 5, 5b = 6, 6c = 7 and 7d = 8. What is  ?
 2 
70. All of David’s telephone numbers have the form 555 – abc – defg, where a, b, c, d, e, f and g are
distinct digits and in increasing order and none is either 0 or 1. How many different telephone numbers
can David have ?
71. For how many positive integers n less than or equal to 24 is n! evenly divisible by 1 + 2 + … + n ?
72. Let x and y be two-digit integers such that y is obtained by reversing the digits of x. Suppose that the
x+y+m
integers x and y satisfy x2 – y2 = m2 for some positive integer ‘m’. What is the value of   ?
 11 
73. A subset ‘B’ of the set of integers from 1 to 100 inclusive, has the property that no two elements of B
sum to 125. What is the maximum possible number of elements in B ?
74. Let n be a 5-digit number, and let q and r be the quotient and remainder, respectively, when n is divided
by 100. Let the total number of value of n for which q + r divisible by 11 is P. Then find the last two
digits of P.
75. Let F(x) denote the sum of the digits of the positive integer x. For example , F(8) = 8 and F(123) = 1 +
2 + 3 = 6. For how many two-digit values of x is F(F(x)) = 3?
76. Given that 38 . 52 = ab, where both a and b are positive integers, find the smallest possible value for
 a + b + 93 
 .
 25 
77. What is the largest integer that is a divisor of
(n + 1) (n + 3) (n + 5) (n + 7) (n + 9)
for all positive even integers n ?
78. The product of three consecutive positive integers is 8 time their sum. What is the sum of their squares?
79. Let P(n) and S(n) demote the product and the sum, respectively, of the digits of the integer n. For
example, P(23) = 6 and S(23) = 5. Suppose N is a two-digit number such that N = P(N) + S(N). What is
the units digit of N ?
80. The mean of three numbers is 10 more than the least of the numbers and 15 less than the greatest. The
median of the three numbers is 5. What is their sum ?
a b
81. + – ab ?
Two non-zero real numbers, a and b, satisfy ab = a – b. Then find the possible value of
b a
82. There are N number of positive integers not exceeding 2001 & are multiples of 3 or 4 but not 5. Then
 N −1
find the value of  .
 50 
83. In the magic square shown, the sums of the numbers in each row, column, and diagonal are the same.
Five of these numbers are represented by v, w, x, y and z. Find y + z.
v 24 w
18 x y
25 z 21
84. A subset of the integers 1, 2, …, 100 has the property that none of its members is 3 times another.
What is the largest number of members such a subset can have ?

H.O. 92, Rajeev Gandhi Nagar, Kota (Raj.) Mob. 97831-97831, 70732-22177, Ph. 0744-2423333 www.nucleuseducation.in 5

5 5
PCCP
85. A positive integer N is a palindrome if the integer obtained by reversing the sequence of digits of N is
equal to N. The year 1991 is the only year in the current century with the following two properties :
(a) It is a palindrome.
(b) It factors as a product of a 2-digit prime palindrome and a 3-digit prime palindrome.
How many years in the millennium between 1000 and 2000 (including the year 1991) have properties
(a) and (b) ?
86. The two-digit intergers from 19 to 92 are written consecutively to form the large integer
N = 19202122….909192
k
If 3 is the highest power of 3 that is a factor of N, then k -
87. For how many n in {1, 2, 3,….. 100} is the tens digit of n2 odd ?
88. If a and b are digits for which
2a
×b 3
6 9
9 2
9 8 9
then a + b =
89. For positive integers n, denote by D(n) the number of pairs of different adjacent digits in the binary
(base two) representation of n. For example, D(3) = D(112) = 0, D(21) = D (101012) = 4, and D(97) =
D(11000012) = 2. For how many positive integers n less than or equal to 97 does D(n) = 2?
90. Call a 7-digit telephone number d1d2d3-d4d5d6d7 memorable if the prefix sequence d1d2d3 is exactly the
same as either of the sequences d4d5d6 or d5d6d7 (possibly both). Assuming that each d1 can be any of
the ten decimal digits 0, 1, 2, … 9. Suppose the total no. of different memorable telephone no.s is ‘n’.
Then find the sum of digits of ‘n’.
91. The number of ordered pairs of integers (m, n) for which mn ≥ 0 and
m3 + n3 + 99mn = 333
is equal to
92. All even numbers from 2 to 98 inclusive, except those ending in 0, are multiplied together. What is the
rightmost digit (the units digit) of the product ?
93. Given that xyz = 2014, and x, y and z are positive integers such that x < y < z, how many possible
triples (x, y, z) are there ?
94. In the following equation, each of the letters represents uniquely a different digit in base ten :
(YE).(ME) = TTT
The sum E + M + T + Y equals
n
2
95. For how many (not necessarily positive) integer values of n is the value of 4000.   an integer ?
5
96. Let S be a set of 6 integers taken from {1, 2, … , 12} with the property that if a and b are elements of S
with a < b, then b is not a multiple of a. What is the least possible values of an element in S ?
97. The total number of non negative integers can be written in the given form are N. Then find sum of all
the digits of N
a7 . 37 + a6 . 36 + a5. 35 + a4 . 34 + a3 . 33 + a2 . 32 + a1 . 31 + a0 . 30,
where a1∈ {–1, 0, 1} for 0 ≤ i ≤ 7 ?
98. Let a, b, c, and d be positive integers such that gcd(a, b) = 24, gcd(b, c) = 36, gcd(c, d) = 54, and 70 <
gcd (d, a) < 100. Which of the following must be a divisor of a?
99. For a positive integer n and nonzero digits a, b, and c, let An be the n-digit integer each of whose digits
is equal to a; let Bn be the n-digit integer each of whose digits is equal to b, and let Cn be the 2n-digit
(not n-digit) integer each of whose digits is equal to c. What is the greatest possible value of a + b + c
for which there are at least two values of n such Cn – Bn = a 2n ?
H.O. 92, Rajeev Gandhi Nagar, Kota (Raj.) Mob. 97831-97831, 70732-22177, Ph. 0744-2423333 www.nucleuseducation.in 6

6 6
PCCP
100. How many ordered pairs (a, b) of positive integers satisfy the equation
a. b + 63 = 20 . lcm(a, b) + 12 . gcd(a, b),
where gcd(a, b) denotes the greatest common divisor of a and b, and lcm(a, b) denotes their least
common multiple ?
101. There are 10 horses, named Horse 1, Horse 2, …, Horse 10. They get their names from how many
minutes it takes them to run one lap around a circular race track : Horse k runs one lap in exactly k
minutes. At time 0 all the horses are together at the starting point on the track. The horses start running
in the same direction, and they keep running around the circular track at their constant speeds. The least
time S > 0, in minutes, at which all 10 horses will again simultaneously be at the starting point is S =
2520. Let T > 0 be the least time, in minutes, such that at least 5 of the horses are again at the starting
point. What is the sum of the digits of T ?
102. How many ordered triples (x, y, z) of positive integers satisfy lcm(x, z) = 600 and lcm(y, z) = 900 ?
x
103. For the greatest value of x, if , 2x is a factor of 101002 – 4501. Find the value of .
15
104. In base 10, the number 2013 ends in the digit 3. In base 9, on the other hand, the same number is
written as (2676)9 and ends in the digit 6. For how many positive integers b does the base-b
representation of 2013 end in the digit 3 ?
105. Real numbers x and y satisfy the equation x2 + y2 = 10x – 6y – 34. What is x + y ?
106. The number 2013 has the property that its units digit is the sum of its other digits, that is 2 + 0 + 1 = 3.
How many integers less than 2013 but greater than 1000 share this property ?
107. The number 2013 is expressed in the form
a !a ...a !
2013 = 1 2 m
b1 !b 2 ...b n !
where a1≥ a2≥ … ≥ am and b1≥ b2≥ … bn are positive integers and a1 + b1 is as small as possible. What is
| a1 – b1 | ?
108. Two integers have a sum of 26. When two more integers are added to the first two integers the sum is
41. Finally when two more integers are added to the sum of the previous four integers the sum is 57.
What is the minimum number of even integers among the 6 integers ?
M 6
109. How many ordered pairs of positive integers (M,N) satisfy the equation = ?
6 N
110. Geeta and Babeeta play the following game. An integer between 0 and 999, inclusive is selsected and
given to Geeta. Whenever Geeta receives a number, he doubles it and passes the result to Babeeta.
Whenever Babeeta receives a number, she adds 50 to it and passes the result to Geeta. The winner is the
last person who produces a number less than 1000. Let N be the smallest initial number that results in a
win for Geeta. What is the sum of the digits of N ?
111. A majority of th 30 students in Mrs. Bhatnagar’s class bought pencils at the school bookstore. Each of
these students bought the same number of pencils, and this number was greater than 1. The cost of a
pencil in paisa was greater than the number of pencils each student bought, and the total cost of all the
pencils was Rs.17.71. What was the cost of a pencil in paisa ?

H.O. 92, Rajeev Gandhi Nagar, Kota (Raj.) Mob. 97831-97831, 70732-22177, Ph. 0744-2423333 www.nucleuseducation.in 7

7 7
PCCP
ANSWER KEY
DPP – 7
Que. 1 2 3 4 5 6 7 8 9 10 11 12
Ans. 3 1 4 3 1 4 2 3 2 1 3 2

13. 05 14. 40 15. 11 16. 75 17. 36 18. 04


19. 86 20. 12 21. 17 22. 35 23. 04 24. 07
25. 60 26. 01 27. 81 28. 16 29. 81 30. 96
31. 49 32. 10 33. 17 34. 18 35. 19 36. 49
37. 13 38. 16 39. 28 40. 34 41. 16 42. 42
43. 03 44. 46 45. 33 46. 21 47. 05 48. 03
49. 07 50. 18 51. 60 52. 50 53. 26 54. 19
55. 81 56. 18 57. 06 58. 18 59. 06 60. 04
61. 04 62. 04 63. 16 64. 45 65. 06 66. 10
67. 05 68. 01 69. 03 70. 08 71. 16 72. 14
73. 62 74. 81 75. 10 76. 20 77. 15 78. 77
79. 09 80. 09 81. 30 82. 02 83. 15 84. 46
85. 76 86. 04 87. 01 88. 20 89. 07 90. 26
91. 28 92. 35 93. 06 94. 05 95. 21 96. 09
97. 04 98. 14 99. 13 100. 18 101. 02 102. 03
103. 15 104. 67 105. 13 106. 2 107. 46 108. 2
109. 01 110. 09 111. 7

H.O. 92, Rajeev Gandhi Nagar, Kota (Raj.) Mob. 97831-97831, 70732-22177, Ph. 0744-2423333 www.nucleuseducation.in 8

8 8
Rourkela Workshop
PRMO
Daily Practice Problems
Sub. : Mathematics DPP : 08

NUMBER SYSTEM
1+ 2 + 3
1. Simplify by rationalizing the denominator.
1− 2 + 3
x 2 − 4x + 3 + (x − 1) x 2 − 9
2. Simplify the expression , where x > 3.
x 2 + 4x + 3 + (x − 1) x 2 − 9
3. Evaluate
( 5+ 6+ 7 )( 5+ 6– 7 )( 5− 6+ 7 ) (− 5+ 6+ 7 . )
4. Suppose that a = 6 – 2 and b = 2 2 – 6 . Then
(A) a > b (B) a = b (C) a < b (D) b = 2a (E) a = 2b
5. Arrange the three values a = 27 – 27 , c = 29 – 28 in ascending order.
26 , b = 28 –
3 3
6. The number of integers x which satisfies the inequality <x< is
1+ 3 5− 3
(A) 2 (B) 3 (C) 4 (D) 5 (E) 6
1 1 2
7. Calculate the value of + + .
1− 5 1+ 5 1+ 5
4 4

8. Given a > b > c > d > 0, and U = ab + cd , V = ac + bd , W = ad + bc . Use “<” to connect


U,V,W.
−1
 4 2 1
9. Simplify 3
3  3 − 3 + 3  .
 9 9 9
3 3 1
10. Given a = 3
4 + 3 2 + 1 , find the value of + + .
a a2 a3
11. Given x + y = 3 5− 2,x–y= 3 2 − 5 , find the value of xy.
12. Evaluate a + 3 + 4 a −1 + a + 3 − 4 a −1 .
( a − 1 − a )5 ( a − 1 + a )5
13. Simplify 3 + 3 .
a −1 + a a − a −1

14. Simplify 1 + a 2 + 1 + a 2 + a 4 .
1 x + 2 + x 2 + 4x
15. Given x = a− , find the value of .
a x + 2 − x 2 + 4x
( )
4
16. Find the last two digits of the greatest integer less than 2 + 3 .
1
17. Consider the function f(x) = . Find the value of
3 + 3
x

3 [f(–5) + f(–4) + f(–3) + f(–2)+ f(–1) + f(0) + f(1) + f(2) + f(3) + f(4) + f(5) + f(6)].
H.O. 92, Rajeev Gandhi Nagar, Kota (Raj.) Mob. 97831-97831, 70732-22177, Ph. 0744-2423333 www.nucleuseducation.in 1

1 1
PCCP
18. How many ordered pairs of positive integers (x, y) satisfy the equation
x y + y x + 2006xy − 2006x − 2006y – 2006= 0?
19. Evaluate X in its simplest form then find sum of all digits of X. Where X is given as

X= 2008 + 2007 2008 + 2007 2008 + 2007 ... .


20. Find the number of ordered pairs of positive integers (x, y) that satisfy the equation
x y + y x + 2009xy − 2009x − 2009y – 2009 = 0.

21. Evaluate 3
77 − 20 13 + 3
77 + 20 13 .

( ) + ( 25 − 10 5 )
1/3 1/3
22. Find the value of 25 + 10 5 .

1 + 2009
23. Let a = . Find the value of (a3 – 503a – 500)5.
2
24. Let n be the positive integer such that
1 1 1 1 1
+ + + ... + =
9 11 + 11 9 11 13 + 13 11 13 15 + 15 13 n n + 2 + (n + 2) n 9

25. If x =
13

2
2009 – 3
1 

2009 
2 3
( )
and y = x + 1 + x , then find the sum of the digits of y.

26. Let x and y be real numbers satisfying


2008x + 2009 2008x + 2009
y= + +25.
2010x − 2011 2011 − 2010x
Find the value of y.
2( 2) + 6)
27. If the value of fraction is equal to 12P. Find P.
3 2+ 3
1 1 1 1 1
28. Let T = − + − + ; then value of T is
3− 8 8− 7 7− 6 6− 5 5−2
29. Suppose that real number x satisfies 49 − x 2 − 25 − x 2 = 3.
What is the value of 49 − x 2 + 25 − x 2 ?
25
30. If the value of 9 − 6 2 + 9 + 6 2 is k, find × k.
6

ANSWER KEY
DPP – 8
x2 − 9
1. 3+ 2 2. 3. 104 4. a > b 5. c < b < a 6. (C). x = 2, 3, 4, 5
x+3
7. –1 8. W < V < U 9. 3
2 +1 10. 1 11. 5– 2
4 if 1 ≤ a < 5 2( a 2 + a + 1 + a 2 − a + 1)
12.   13. 4 a(a − 1) 14. 15. a2
2 a − 1 if a ≥ 5  2
16. 93 17. 06 18. 08 19. 10 20. 06 21. 07
22. 05 23. 32 24. 79 25. 11 26. 25 27. 16
28. 05 29. 08 30. 50

H.O. 92, Rajeev Gandhi Nagar, Kota (Raj.) Mob. 97831-97831, 70732-22177, Ph. 0744-2423333 www.nucleuseducation.in 2

2 2
Rourkela Workshop
PRMO
Daily Practice Problems
Sub. : Mathematics DPP : 09

NUMBER SYSTEM

1. Prove that 7 | (22225555 + 55552222).


27
2. Find the remainder of 4737 when it is divided by 11.
3. What is the remainder when 91990 is divided by 11 ?
4. When a positive integer n is divided by 5, 7, 9, 11, the remainders are 1, 2, 3, 4respectively. Find the
minimum value of n.
5. Find the last two digits of 2999.
6. Find the remainder of (25733 + 46)26 when it is divided by 50.
7. What is the smallest positive integer n > 1 such that 3n ends with 003 ?
8. Find all the positive integers n, such that n.2n + 1 is divisible by 3.

9. Prove that when abc is a multiple of 37, then so is the number bca .
10. Find all positive integers with initial digit 6 such that the integer formed by deleting this 6 is 1/25 of the
original integer.
11. Let x be a 3-digit number such that the sum of the digits equals 21. If the digits of x are reversed, the
number thus formed exceeds x by 495. What is x ?
12. Given that a four digit number n and the sum of all digits of n have a sum 2001. Find n.
13. When N = 11...11
 × 11...11
 , what is the sum of all digits of N?
1989 digits 1989 digits

14. Find the maximum value of the ratio of three digit number to the sum of its digits.

15. Find all the three digit numbers n = abc such that n = (a + b + c)3.
16. Given that the natural numbers numbers a, b, c are formed by the same n digits x, n digits y, and 2n
digits z respectively. For any n ≥ 2 find the digits x, y, z such that a2 + b = c.
17. When a two digit number is divided by the number formed by exchanging the two digits, the quotient is
equal to its remainder. Find the two digit number.
18. Find the maximum perfect square, such that after deleting its last two digits (which is assumed to be not
all zeroes), the remaining part is still a perfect square.

H.O. 92, Rajeev Gandhi Nagar, Kota (Raj.) Mob. 97831-97831, 70732-22177, Ph. 0744-2423333 www.nucleuseducation.in 1

1 1
PCCP

ANSWER KEY

DPP – 9

2. 9 3. 1 4. 1731 5. 88
6. 29 7. 101 8. all numbers in the form 6k + 1 or 6k +2, k = 0, 1, 2,...
10. 625 × 10n – 2 (n ≥ 2) 11. 489 12. 1977 13. 17901
15. 512 16. (3,2,1), (6,8,4), (8,3,7) 17. n = 52 18. 1681

H.O. 92, Rajeev Gandhi Nagar, Kota (Raj.) Mob. 97831-97831, 70732-22177, Ph. 0744-2423333 www.nucleuseducation.in 2

2 2
Rourkela Workshop
PRMO
Daily Practice Problems
Sub. : Mathematics DPP : 10

NUMBER SYSTEM

1. If (x – 1) (x – 4) (x – 8) + m is a perfect square, then m is


(1) 32 (2) 24 (3) 98 (4) 196

2. If n + 20 and n – 21 are both perfect squares, where n is a natural number, find n.

3. Find the maximal integer x such that 427 + 410000 + 4x is a perfect square.

4. Prove that for any positive integer n, n4 + 2n3 + 2n2 + 2n + 1 is not a perfect square.

5. Prove that there is no three digit number abc , such that abc + bca + cab is a perfect square.

6. Prove that the equation a2 + b2 – 8c = 6 has no integer solution.

7. Given that p is a prime number, and the sum of all positive divisors of p4 is a perfect square. Find the
number of such primes p.

8. If x and y are positive integers, prove that the values of x2 + y + 1 and y2 + 4x + 3 cannot both be
perfect squares at the same time.

9. Let d be any positive integer not equal to 2, 5, or 13. Show that one can find distinct a, b in the set {2, 5,
13, d} such that ab – 1 is not a perfect square.

10. Prove that the number 3n + 2 × 17n, where n is a non-negative integer, is never a perfect square.

11. Let p be a prime number such that the next larger number is a perfect square. Find the sum of all such
prime numbers. (For example, if you think that 11 and 13 are two such prime numbers, then the sum is
24.)

12. The value of greatest integer N such that


N ≤ 2007 2 – 20070 + 31 .
N−2
Find
100

H.O. 92, Rajeev Gandhi Nagar, Kota (Raj.) Mob. 97831-97831, 70732-22177, Ph. 0744-2423333 www.nucleuseducation.in 1

1 1
PCCP
13. Find the sum of all the digit of largest integer N such that both N + 496 and N + 224 are perfect
squares.

14. The sum of 18 consecutive positive integers is a perfect square. In the smallest possible value of this
sum, find the sum of first two integers ?

15. Let n be the smallest positive integer such that n is divisible by 20, n2 is a perfect cube, and n3 is a
perfect square. The number n contains N digits. What is N2?

16. How many positive integers n are there such that 7n + 1 is a perfect square and 3n + 1 < 2008?

17. The product 1 × 2 × 3 × … × n is denoted by n!. For example 4! = 1 × 2 × 3× 4 = 24. Let M = 1! × 2! ×


3! × 4! × 5! × 6! × 7! × 8! × 9!. Now the number of factors of M are perfect squares is (21b). Then find
the value of b.

18. Find the number of ordered pairs (x, y), where x is an integer and y is a perfect square, such that
y = (x – 90)2 – 4907.

19. The number 2564. 6425 is the square of a positive integer N. In decimal representation, the sum of the
digits of N is.

n
20. For how many integers n is the square of an integer ?
20 − n

21. If 554 is the base b representation of the square of the number whose base b representation is 24, then b,
when written in base 10, equals

ANSWER KEY
DPP – 10
1. (4) 2. 421 3. 1972 7. p = 3 (unique solution) 11. 03
12. 20 13. 17 14. 09 15. 49 16. 18 17. 32
18. 04 19. 14 20. 04 21. 12

H.O. 92, Rajeev Gandhi Nagar, Kota (Raj.) Mob. 97831-97831, 70732-22177, Ph. 0744-2423333 www.nucleuseducation.in 2

2 2
Rourkela Workshop
PRMO
Daily Practice Problems
Sub. : Mathematics DPP : 11

NUMBER SYSTEM

1. If n is a positive integer, prove that  n + n + 1  =  4n + 2  .


2. Solve equation [x3] + [x2] + [x] = {x} – 1.
1
3. Solve equation [3x + 1] = 2x – , and find the sum of all roots.
2
10n 
4. Find the minimum natural number n, such that the equation   = 1989 has integer solution x.
 x 
5. How many of the first 1000 positive integers can be expressed in the form [2x] + [4x] + [6x] + [8x],
where x is a real number, and [z] denotes the greatest integer less than or equal to z ?
6. How many different non-negative integers are there in the sequence
 12   22   32  19802 
  
,  
,  ,...,   ?
1980  1980  1980   1980 
7. For every positive integer n, show that  n + n + 1  =  4n + 1  =  4n + 2  =  4n + 3  .
8. Solve the equation [x] {x} + x = 2{x} + 10.
 1093 
9. Find the last two digits of the number  31  (Write down the tens digit first, then write down the
10 + 3 
units digit).
92 92
10. Solve the equation x + = [x] + .
x [x]
11. If x is a real number that satisfies
 11   12   99 
 x + 100  +  x + 100  + ... +  x + 100  = 765,
     
find the value of [900] – [100x]. Here [a] denotes the largest integer ≤ a.
12. Find the number of consecutive 0’s at the end of the base 10 representation of 206!.
n  n  n 
13. There are P number of positive integer n less than 2007 such that   +   +   = n where [x] is the
2 3 6
greatest integer less than or equal to x. Write the last two digit of P.
(For example, [2.5] = 2; [5] = 5; [–2.5] = –3 etc.)
14. Let [x] denote the greatest integer less than or equal to x. Some P real values of x satisfy the equation x2
+ 10,000 [x] = 10,000x. What is P – 100 ?

H.O. 92, Rajeev Gandhi Nagar, Kota (Raj.) Mob. 97831-97831, 70732-22177, Ph. 0744-2423333 www.nucleuseducation.in 1

1 1
PCCP
15. For any real number x, let  x  denote the smallest integer that is greater than or equal to x and  x 
denote the largest integer that is less than or equal to x (for example, 1.23 2 and 1.23 = 1). Find the
2010
 2010  2010  
value of 2010 – ∑
k =1  k
−
 k 
 .

16. Given that


x = [11/3] + [21/3] + [31/3] + … + [79991/3],
 x 
find the value of   , where [y] denotes the greatest integer less than or equal to y. (For example,
 5000 
[2.1] = 2, [30] = 30, [– 10.5] = – 11.)
3100 + 2100
17. What is the greatest integer less than or equal to ?
396 + 296
18. Let [x] denote the greatest integer less than or equal to x. If there are N values of x satisfy the equation
N +1
x2 + 10,000[x] = 10,000x. Then find the value of .
4


10
19. Let f(x) = k =2
([kx] − k[x]) , where [r] denotes the greatest integer less than or equal to r. How many
distinct values does f(x) assume for x ≥ 0 ?

ANSWER KEY

DPP – 11
11. 47 12. 50 13. 34 14. 99 15. 16 16. 23
17. 80 18. 50 19. 32

H.O. 92, Rajeev Gandhi Nagar, Kota (Raj.) Mob. 97831-97831, 70732-22177, Ph. 0744-2423333 www.nucleuseducation.in 2

2 2
Rourkela Workshop
PRMO
Daily Practice Problems
Sub. : Mathematics DPP : 12
GEOMETRY
1. ABC is an isosceles triangle with AB = AC = 2008 cm. ADC is drawn as an equilateral triangle on AC
outside ∆ABC. AD is parallel to BC. The bisector of D meets AB in E, say. Then BE is equal to
(1) 1004 cm (2) 2008 cm (3) 0 (4) 502 cm
2. In the adjoining figure ABC, DEF are equilateral triangles AB = 8 cm and DE = 3cm. Then the possible
value of AE + BD + CF is

(1) 6.9 cm (2) 7.1 cm (3) 5.2 cm (4) 8.3 cm


3. The measure of one of the angles of a right triangle is five times that of a second angle. Then the
possibility of the second largest angle is
(1) 72° (2) 7.5° (3) 72° or 75° (4) None of these
4. Two of the altitudes of the scalene triangle ABC have lengths 4 and 12. If the length of the third altitude
is also an integer, then its greatest value can be ..... . .
(1) 5 (2) 10 (3) 15 (4) 20
5. The perimeter of an isosceles right angled triangle is 2012. Its area is
(1) 2012(3 – 2 ) (2) (1006)2 (3 – 2 2 ) (3) (2012)2 (4) (1006)2
6. Triangles ABC and ABD are isosceles with AB = AC = BD, and AC intersects BD at E. If AC is
perpendicular to BD, then ∠C + ∠D is
(1) 115° (2) 120° (3) 130° (4) 135°
7. In ∆ABC, AB = 8, BC = 7, CA = 6. Extend BC to P such that ∆PAB ∆ ∆PCA, then the length of PC is
(1) 7, (2) 8, (3) g, (4) 10, (E) 11
8. In the isosceles right triangle ABC, ∠B = 90º, AD is the median on BC. Write AB = BC = a. If BE ⊥
AD, intersecting AC at E, and EF ⊥ BC at F, then EF is
1 1 2 2
(1) a, (2) a, (3) a, (4) a.
3 2 3 5
9. In the adjoining figure BAC is a 30º – 60° – 90º triangle with AB = 20. D is the midpoint of AC. The
perpendicular at D to AC meets the line parallel to AB through C at E. The line through E
perpendicular to DE meets BA produced at F. If DF = 5 x , then find x.

(1) 1 (2) 7 (3) 5 (4) 2

H.O. 92, Rajeev Gandhi Nagar, Kota (Raj.) Mob. 97831-97831, 70732-22177, Ph. 0744-2423333 www.nucleuseducation.in 1

1 1
PCCP
10. A point P is chosen in the interior of ∆ABC such that when lines are drawn through P parallel to the
sides of ∆ABC, the resulting smaller triangles t1, t2, and t3 in the figure, have areas 4, g and 49
respectively. Find the area of ∆ABC.
(1) 144 (2) 140 (3) 130 (4) 131
11. In ∆ABC, D, E, F are on the sides BC, CA, AB respectively, such that they are concurrent at a point G,
BD = 2CD, the areas S1 = [GEC] = 3, S2 = [GCD] = 4. And the area of ∆ABC.
(1) 30 (2) 20 (3) 25 (4) 35
12. In ∆ABC, the bisector of ∠A intersects BC at D. A perpendicular to AD from B intersects AD at E. A
line segment through E and parallel to AC intersects BC at G, and AB at H. If AB = 26, BC = 28, AC
= 30, find the measure of DG .

(1) 2 (2) 3 (3) 1 (4) 4


( AE )2 + ( BF )2
13. If AE and BF are medians drawn to the legs of right ∆ABC, find the numerical value of
( AB)2

5 6 7 8
(1) (2) (3) (4)
4 4 4 4
14. The measures of the bases of trapezoid ABCD are 15 and 9, and the measure of the altitude is 4. Legs
DA and CB are extended to meet at E. If F is the midpoint of AD , and G is the midpoint of BC , find
the area of ∆FGE. (The figure is not drawn to scale.)

(1) 48 (2)49 (3) 46 (4) 52


15. In a right-angled ∆ABC, ∠ACB = 90°, E, Fare on AB such that AE = AC, BF = BC, find ∠ECF in
degrees.
16. Given that the lengths of three sides, a, b, c of a triangle are positive integers, where a < b < c. Find the
number of the triangles with b = 2.
17. In a right-angled triangle, if the length of a leg is 21, and the lengths of the other two sides are also
positive integers, find the minimum value of its possible perimeter.
18. In right-angled ∆ABC, ∠C = 90°, E is on BC such that AC = BE. D is on AB such that DE ⊥ BC.
1
Given that DE + BC = 1, BD = , find ∠B in degrees.
2
19. In ∆ABC, AC = BC, ∠C = 20º, M is on the side AC and N is on the side BC, such that ∠BAN = 50°,
∠ABM = 60°. Find ∠NMB in degrees.
20. In the ∆ABC, BE is the angle bisector of the ∠ABC, AD is the median on the side BC, and AD
intersects BE at O perpendicularly. Given BE =AD = 4, find the lengths of three sides of ∆ABC.

H.O. 92, Rajeev Gandhi Nagar, Kota (Raj.) Mob. 97831-97831, 70732-22177, Ph. 0744-2423333 www.nucleuseducation.in 2

2 2
PCCP
21. In ∆ABC, D is the midpoint of BC, E is on AC such that AC = 3EC. BE and AD intersect at G. Find
AG : GD.
22. When extending the AB, BC, CA of ∆ABC to B', C', A' respectively, such that AB' = 2AB, CC' = 2BC,
AA' = 3CA. If area of ∆ABC is l, find the area of ∆A'B'C.
23. Given that the point P is outside the equilateral triangle ABC but inside the region of ∠ABC. If the distances
from P to BC, CA, AB are h1, h2 and h3 respectively, and h1 – h2 + h3 = 6, find the area of ∆ABC.
24. Given that G is the centroid of ∆ABC, GA = 2 3 , GB = 2 2 , GC = 2. Find the area of ∆ABC.
25. Point P is inside ∆ABC. Line segments APD, BPE, and CPF are drawn with D on BC, E on CA, and F on
AB (see the figure below). Given that AP = 6, BP = 9, PO= 6, PE = 3, and CF = 20, find the area of ∆ABC.

26. In a square ABCD, let P be a point on the side BC such that BP = 3PC and Q be the mid-point of CD. If
the area of the triangle PCQ is 5, what is the area of triangle QDA?

27. A triangle ∆ABC is inscribed in a circle of radius 4 cm. Suppose that ∠A = 60°, AC – AB = 4 cm, and
2
x
the area of ∆ABC is x cm2. Find the value of   .
2
28. In ∆ABC, AB : AC = 4 : 3 and M is the midpoint of BC. E is a point on AB and F is a point on AC
such that AE : AF = 2 : 1. It is also given that EF and AM intersect at G with GF = 36 cm and GE = x
cm. Find the value of x.

29. Non-degenerate ∆ABC has integer side lengths, BD is an angle bisector, AD = 3, and DC = 8. What is
the smallest possible value of the perimeter?
30. It is known that the 3 sides of a triangle are consecutive positive integers and the largest angle is twice
the smallest angle. Find the perimeter of this triangle.
31. A triangle ∆ABC is inscribed in a circle of radius 1, with ∆BAC = 60°. Altitudes AD and BE of ∆ABC
intersect at H. Find the smallest possible value of the length of the segment AH.
32. In triangle ABC, AB = 28, BC = 21 and CA = 14. Points D and E are on AB with AD = 7 and ∠ACD =
∠BCE. Find the length of BE.
33. Four points in the order A, B, C, D lie on a circle with the extension of AB meeting the extension of DC at E
and the extension of AD meeting the extension of BC at F. Let EP and FQ be tangents to this circle with
points of tangency P and Q respectively. Suppose EP = 60 and FO = 63. Determine the length of EF.
34. The diagram shows an equilateral triangle ADE inside a square ABCD. What is the value of
2
 area of ∆ADE 
  .
 area of ∆ADE 

H.O. 92, Rajeev Gandhi Nagar, Kota (Raj.) Mob. 97831-97831, 70732-22177, Ph. 0744-2423333 www.nucleuseducation.in 3

3 3
PCCP

35. In this question, S∆xyz denotes the area of ∆XYZ. In the following figure, if DE || BC, S∆ADE = 1 and S∆ADC
= 4, find S∆DBC.

36. ∆ABC is a right-angled triangle with ∠ABC = 90°. A circle C1 is drawn with AB as diameter, and
another circle C2 is drawn with BC as diameter. The circles C1 and C2 meet at the points B and P. If AB
120
= 5 cm, BC = 12 cm and BP = x cm, find the value of .
x

1
37. In the following diagram, ∠ACB = 90°, DE ⊥ BC, BE = AC, BD = cm, and DE + BC = 1 cm.
2
Suppose ∠ABC = xº. Find the value of x.

38. In the figure below, S is a point on OR and U is a point on PR. The line segments PS and QU intersect
at the point T. It is given that PT = TS and QS = 2RS. If the area of ∆PQR is 150 cm2 and the area of
∆PSU is x cm2. Find the value of x.

39. A triangle ∆ABC has its vertices lying on a circle C of radius 1, with ∠BAC = 60°. A circle with I is
inscribed in ∆ABC. The line AI meets circle C again at D. Find the length of the segment ID.
40. In the triangle ABC, AB = AC = 1, D and E are the midpoints of AB and AC respectively. Let P be a
point on DE and let the extensions of BP and CP meet the sides AC and AB at G and F respectively.
1 1
Find the + .
BF CG

H.O. 92, Rajeev Gandhi Nagar, Kota (Raj.) Mob. 97831-97831, 70732-22177, Ph. 0744-2423333 www.nucleuseducation.in 4

4 4
PCCP

41. In the triangle ABC, AB = 14, BC = 16, AC = 26, M is the midpoint of BC and D is the point on BC
such that AD bisects ∠BAC. Let P be the foot of the perpendicular from B onto AD. Determine the
length of PM.

42. In the triangle ABC, ∠A = 20°, ∠C = 90º, 0Ois a point on AB and D is the midpoint of OB. Suppose the
circle centered at O with radius OD touches the side AC at T. Determine the size of ∠BCD in degrees.

43. In ∆ABC, D and E are the midpoints of the sides AB and AC respectively, CD and BE intersect at P
with ∠BPC = 90°. Suppose BD = 8 5 and CE = 6 5 . Find BC.

44. In the following right-angled triangle ABC, AC = BC = 1 and DEF is an arc of a circle with center A.
x
Suppose the shaded areas BDE and CEF are equal and AD = . What is the value of x?
π

45. The diagram below shows a triangle ABC in which ∠A = 60°, BP and BE trisect ∠ABC and CP and
CE trisect ∠ACB. Let the angle ∠BPE be xº. Find x.

H.O. 92, Rajeev Gandhi Nagar, Kota (Raj.) Mob. 97831-97831, 70732-22177, Ph. 0744-2423333 www.nucleuseducation.in 5

5 5
PCCP

46. In ∆ABC ∠BAC = 45°, D is a point on BC such that AD is perpendicular to BC. If BD = 3 cm and
DC= 2 cm, and the area of the ∆ABC is x cm2. Find the value of x.

47. In ∆ABC (see below), AB = AC = 3 and D is a point on BC such that AD = 1. Find the value of BD · DC.
48. In ∆ABC, ∠CAB = 30° and ∠ABC = 80°. The point M lies inside the triangle such that ∠MAC = 10º
and ∠MCA = 30°. Find the value of (180° – ∠BMC) in degrees.
49. Let ABC be any triangle. Let D and E be the points respectively in the segments of AB and BC such
that AD = 7DB and BE = 10EC. Assume that AE and CD meet at point F. Determine [k], where k is the
real number such that AF = k x FE.
50. In an acute-angled triangle ABC, points D, E and F are the feet of the perpendiculars from A, B, and C
5
onto BC, AC and AB respectively. Suppose sin A= and BC = 39, find the length of AH, where H is
3
the intersection AD with BE.
51. Let O be the centre of the circum circle of ∆ABC, P and Q the midpoints of AO and BC, respectively.
Suppose ∠CBA = 4∠OPQ and ∠ACB = 6∠OPQ. Find the size of ∠OPQ in degrees.
52. In ∆ABC, AC > AB, the internal angle bisector of ∠A meets BC at D, and E is the foot of the
perpendicular from B onto AD. Suppose AB = 5, BE = 4 and AE = 3. Find the value of the expression
 AC + AB 
 AC − AB  ED.
 
53. Let O be an interior point of ∆ABC. Extend AO to meet the side BC at D. Similarly, extend BO and CO to
meet CA and AB respectively at E and F. If AO = 30, FO = 20, BO = 60, DO = 10 and CO = 20, find EO.
54. In ∆ABC, AB = 13, AC = 5, and BC = 12. Points M and N lie on AC and BC , respectively, with CM
= CN = 4. Points J and K are on AB so that MJ and NK are perpendicular to AB . What is the whole
part of area of pentagon CMJKN

55. In triangle ABC, side AC and the perpendicular bisector of BC meet in point D, and BD bisects ∠ABC.
If AD= 9 and DC = 7, the area of triangle ABD = a 5 then the value of a?
56. In ∆ABC, we have AB = 1 and AC = 2. Side BC and the median from A to BC have the same length. If
the length of BC is a then the value of a?
H.O. 92, Rajeev Gandhi Nagar, Kota (Raj.) Mob. 97831-97831, 70732-22177, Ph. 0744-2423333 www.nucleuseducation.in 6

6 6
PCCP
57. Triangle ABC with AB = 50 and AC = 10 has area 120. Let D be the midpoint of AB , midpoint of AC
. The angle bisector of ∠BAC intersects DE and BC at F and G, respectively. What is the area of
quadrilateral FDBG?
58. Triangle ABC is an isosceles right triangle with AB = AC = 3. Let M be the midpoint of hypotenuse BC .
Points I and E lie on sides AC and AB, respectively, so that AI > AE and AIME is a cyclic quadrilateral.
a− b
Given that triangle EMI has area 2, the length CI can be written as , where a, b and c are positive
c
integers and be is not divisible by the square of any prime. What is the value of a + b + c?
59. In ∆PAT, ∠P = 36°, ∠A = 56°, and PA = 10. Points U and G lie on sides TP and TA, respectively, so
PU = AG = 1. Let M and N be the midpoints of segments PA and UG , respectively. What is the
degree measure of the acute angle formed by lines MN and PA?
60. Let ABC be an equilateral triangle. Extend side AB beyond B to a point B' so that BB' = 3AB.
Similarly extend side BC beyond C to a point C' so that CC' = 3 BC, and extend side CA beyond A to
a point A' so that AA' = 3CA. The ratio of the area of ∆A'B'C' to the area of ∆ABC is Q : 1. What is Q?
61. In ∆ABC, AB = 6, BC = 7, and CA = 8. Point D lies on BC , and AD bisects ∠BAC. Point E lies on
AC , and BE bisects ∠ABC. The bisectors intersect at F. The ratio of AF: FD is C : 21. What is C?

62. In ∆ABC shown in the figure, AB = 7, BC = 8, CA = 9, and AH is an altitude. Points D and E lie on
sides AC and AB respectively, so that BD and CE are angle bisectors, intersecting AH at Q and P,
a
respectively. If PQ = c , where a, b are co-prime, then what is the value of (a+ b + c2)?
b

63. There are f non-congruent integer-sided triangles with positive area and perimeter less than 15, which
are neither equilateral, isosceles, nor right triangles. What is 7f?
64. In ∆BAC, ∠BAC = 40°, AB = 10, and AC = 6. Points D and E lie on AB and AC respectively. What
is the minimum possible value of BE + DE + CD?
65 . In ∆ABC, AB = AC = 28 and BC = 20 Points D, E and F are on sides AB , BC , and AC , respectively,
such that DE and EF are parallel to AC and AB , respectively. What is the perimeter of
parallelogram ADEF?

H.O. 92, Rajeev Gandhi Nagar, Kota (Raj.) Mob. 97831-97831, 70732-22177, Ph. 0744-2423333 www.nucleuseducation.in 7

7 7
PCCP

66. Triangle ABC is equilateral with AB = 1. Points E and G are on AC and points D and F are on AB
such that both DE and FG are parallel to BC . Furthermore, triangle ADE and trapezoids DFGE and
r
FBCG all have the same perimeter. DE + FG can be written as as r and s are co-prime. What is r + s?
s

67. The angles in a particular triangle are in arithmetic progression, and the side lengths are 4, 5, x. The sum of
the possible values of x equals a + b + c where a, b, and c are positive integers. What is a + b + c?
68. In triangle ABC, AB = 13, BC = 14 and CA = 15. Distinct points D, E, and F lie on segments BC , CA
and DE , respectively, such that AD ⊥ BC , DE ⊥ AC , and AF ⊥ BF . The length of segment DF
m
can be written as , where m and n are relatively prime positive integers. What is m + n?
n
69. Triangle ABC has AB = 27, AC = 26, and BC = 25. Let I denote the intersection of the internal angle
bisectors of ∆ABC. What is BI?
70. Triangle ABC has side-lengths AB = 12, BC = 24, and AC = 18. The line through the incenter of
∆ABC parallel to BC intersects AB at M and AC at N. What is the perimeter of ∆AMN?
71. Points B and C lie on AD . The length of AB is 4 times the length of BD , and the length of AC is 9
times the length of CD . The length of BC × S = the length of AD . What is S?
72. Point P is inside equilateral ∆ABC. Points Q, R and S are the feet of the perpendiculars from P to AB ,
BC and CA , respectively. Given that PQ = 1, PR = 2, and PS = 3, what is AB2?
73. In triangle ABC, AB = (b2 – 1) cm, BC = a2 cm and AC = 2a cm, where a and b are positive integers
greater than 1. And the value of a - b.
74. In the figure below, ADE is a triangle with ∠AED = 120°, and B and C are points on side AD such that
BCE is an equilateral triangle. If AB = 4 cm, CD = 16 cm and BC = x cm, find the value of x.

75. The figure below shows a triangle ABC where AB = AC. D and E are points on sides AB and AC,
respectively, such that AB = 4DB and AC = 4AE. If the area of the quadrilateral BCED is 52 cm2 and
the area of the triangle ADE is x cm2, find x.

H.O. 92, Rajeev Gandhi Nagar, Kota (Raj.) Mob. 97831-97831, 70732-22177, Ph. 0744-2423333 www.nucleuseducation.in 8

8 8
PCCP

76. In the triangle ABC, AC = 2BC, LC = 90º and D is the foot of the altitude from C onto AB. A circle
with diameter AD intersects the segment AC at E. Find AE : EC.
77. In the triangle ABC, AB = 8, BC = 7 and CA = 6. Let E be the point on BC such that ∠BAE = 3∠EAC.
4AE 2
Find .
5
78. In the triangle ABC, the bisectors of ∠A and ∠B meet at the incentre I, the extension of AI meets the
circum circle of triangle ABC at D. Let P be the foot of the perpendicular from B onto AD, and Q a
point on the extension of AD such that ID = DQ. Determine the value of (BQ × IB)/(BP × ID).
79. Three sides OAB, OAC and OBC of a tetrahedron OABC are right-angled triangles, i.e. ∠AOB =
∠AOC = ∠BOC = 90º. Given that OA = 7, OB = 2 and OC = 6, Find the value of
[(Area of ∆OAB)2 + (Area of ∆OAC)2 + (Area of ∆OBC)2 + (Area of ∆ABC)2 + 37]1/2.
80. In a triangle ABC, the length of the altitudes AD and BE are 4 and 12 respectively.
Find the largest possible integer value for the length of the third altitude CF.
81. In the figure below, ABC is a triangle and D is a point on side BC Point E is on side AB such that DE is
the angle bisector of ∠ADB, and point F is on side AC such that DF is the angle bisector of ∠ADC.
AE BD CF
Find the value of . . .
EB DC FA

82. In the figure below, ABC is a triangle with AB = 10 cm and BC = 40 cm. Points D and E lie on side AC
and point F on side BC such that EF is parallel to AB and OF is parallel to EB. Given that BE is an
angle bisector of ∠ABC and that AD = 13.5 cm, find the length of CD in cm.

83. In triangle ABC, AB = 13, BC = 14 and CA = 15. Distinct points D, E and F lie on segments BC , CA
and DE , respectively, such that AD ⊥ BC , DE ⊥ AC and AF ⊥ BF . The length of segment DF can
m
be written as , where m and n are relatively prime positive integers. What is m + n?
n

84. In ∆ABC, medians AD and CE intersect at P, PE = 1.5, PD = 2 and DE = 2.5. What is the area of 2[AEDC]?

H.O. 92, Rajeev Gandhi Nagar, Kota (Raj.) Mob. 97831-97831, 70732-22177, Ph. 0744-2423333 www.nucleuseducation.in 9

9 9
PCCP

85. Triangle ABC has AB = 2 . AC. Let D and E be on AB and BC , respectively, such that ∠BAE =
∠ACD. Let F be the intersection of segments AE and CD, and suppose that ∆CFE is equilateral. What
is ∠ACB (in degrees)?
86. A flagpole is originally 5 meters tall. A hurricane snaps the flagpole at a point x meters above the
ground so that the upper part, still attached to the stump, touches the ground 1 meter away from the
base. What is [x] where [:] is the greatest integer function?
87. Distinct points A, B, C, and D lie on a line, with AB = BC = CD= 1. Points E and F lie on a second line,
parallel to the first, with EF = 1. A triangle with positive area has three of the six points as its vertices.
How many possible values are there for the area of the triangle?
88. An equilateral triangle has side length 6. What is the area of the region containing all points that are outside
the triangle and not more than 3 units from a point of the triangle to the nearest integer? (Take π = 3.14)
89. Two equilateral triangles are contained in a square whose side length is 2 3 . The bases of these
triangles are the opposite sides of the square and their intersection is a rhombus. If the area of the
rhombus is 8 k – 12, find k.
90. A right triangle has perimeter 32 and area 20. What is the length of its hypotenuse to the nearest integer?
91. A triangle ∆ABC is inscribed in a circle of radius 1, with ∠BAC = 60°. Altitudes AD and BE of ∆ABC
intersect at H. Find the smallest possible value of the length of the segment AH.
92. Two sides of a triangle have lengths 10 and 15. The length of the altitude to the third side is the average
of the lengths of the altitudes to the two given sides. How long is the third side?
93. In ∆ABC, AB = AC = 28 and BC = 20. Points D, E and F are on sides AB , BC , and AC respectively, such
that DE and EF are parallel to AC and AB , respectively. What is the perimeter of parallelogram ADEF?

94. In ∆ABC, ∠C = 90º and AB = 12. Squares ABXY and ACWZ are constructed outside of the triangle.
The points X, Y, Z and W lie on a circle. The perimeter of the triangle can be written as a(1 + 2 ) .
Find the product of a and b.
95. Point P is inside equilateral ∆ABC. Points Q, R and S are the feet of the perpendiculars from P to AB .
BC and CA , respectively. Given that PQ = 1, PR = 2, and PS = 3, what is AB2?
96. Right ∆ABC has AB = 3, BC = 4, and AC = 5. Square XY ZW is inscribed in DABC with X and Y on
AC , W on AB , and Z on BC . What is the side length of the square to the nearest integer?

97. In a triangle with integer side lengths, one side is three times as long as a second side, and the length of
the third side is 15. What is the greatest possible perimeter of the triangle?
H.O. 92, Rajeev Gandhi Nagar, Kota (Raj.) Mob. 97831-97831, 70732-22177, Ph. 0744-2423333 www.nucleuseducation.in 10

10 10
PCCP
98. A triangle is partitioned into three triangles and a quadrilateral by drawing two lines from vertices to
their opposite sides. The areas of the three triangles are 3, 7, and 7, as shown. What is the area of the
shaded quadrilateral?

99. Equilateral ∆ABC has side length 2, M is the midpoint of AC , and C is the midpoint of BD . If the area
of ∆CDM is 'x', then find the value of 4x2.

100. In right triangle ∆ACE, we have AC = 12, CE = 16, and EA = 20. Points B, D, and F are located on AC
, CE , and EA , respectively, so that AB = 3, CD = 4, and EF = 5. What is the difference of the area of
∆ACE to that of ∆DBF?

101. In ∆ABC points D and E lie on BC and AC , respectively. If AD and BE intersect at T then AT/DT =
 11CD 
3 and BT/ET = 4. What is  ?
 BD 

102. A triangle with sides of 5, 12, and 13 has both an inscribed and a circumscribed circle. Find square of
twice the distance between th8 centres of those circles?
103. A large equilateral triangle is constructed by using toothpicks to create rows of small equilateral
triangles. For example, in the figure we have 3 rows of small congruent equilateral triangles, with 5
small triangles in the base row. Total N number of toothpicks would be needed to construct a large
equilateral triangle if the base row of the triangle consists of 2003 small equilateral triangles. Then find
the sum of all digits of N.

H.O. 92, Rajeev Gandhi Nagar, Kota (Raj.) Mob. 97831-97831, 70732-22177, Ph. 0744-2423333 www.nucleuseducation.in 11

11 11
PCCP

104. The sides of a triangle have lengths of 15, 20, and 25. Find the length of the shortest altitude.
105. Points A,B,C,D,E and F lie, in that order, on AF , dividing it into five segments, each of length 1. Point
G is not on line AF. Point H lies on GD , and point J lies on GF . The line segments HC , JE , and AG
 3HC 
are parallel. Find  .
 JE 
106. Points A,B,C and D lie on a line, in that order, with AB = CD and BC = 12. Point E is not on the line,
and BE = CE = 10. The perimeter of ∆AED is twice the perimeter of ∆BEC. Find AB.
107. Let ∆XOY be a right-angled triangle with ∠XOY = 90°. Let M and N be the midpoints of legs OX and
OY, respectively. Given that XN = 19 and YM = 22, find XY.
108. In ∆ABC, ∠A = 100°, ∠B = 50°, ∠C = 30°, AH is an altitude and BM is a median. Then ∠MHC (in
degrees) is

109. Triangle ABC has a right angle at c, AC = 3 and BC = 4. Triangle ABD has a right angle at A and AD
= 12. Points C and D are on opposite sides of AB . The line through D parallel to AC meets CB
DE m m+n
extended at E. If = where m and n are relatively prime positive integers, then is
DB n 2

110. Equilateral triangle ABC has been creased and folded so that vertex A now rests at A' on BC as shown.
k 21
If BA' = 1 and A'C = 2. If the length of crease PQ is , find k.
20

111. In triangle ABC, ∠ABC = 120°, AB = 3 and BC = 4. If perpendiculars constructed to AB at A and to


BC at C meet at D, then 3 CD equals?
112. Triangle ABC and point Pin the same plane are given. Point P is equidistant from A and B, angle APB
is twice angle ACB, and AC intersects BP at point D. If PB = 3 and PD = 2, then AD·CD =
H.O. 92, Rajeev Gandhi Nagar, Kota (Raj.) Mob. 97831-97831, 70732-22177, Ph. 0744-2423333 www.nucleuseducation.in 12

12 12
PCCP

113. In triangle ABC, AB = 13, BC = 14, and AC = 15. Let D denote the midpoint of BC and let E denote
the intersection of BC with the bisector of angle BAC. Which of the following is closest to the area of
the triangle A DE?
114. Two congruent 30° – 60° – 90° triangles are placed so that they overlap partly and their hypotenuses
coincide. If the hypotenuse of each triangle is 12, and the area common to both triangles is given by A.
Find the value of 3 A·
115. In ∆ABC with right angle at C, altitude CH and median CM trisect the right angle. If the area of ∆CHM
is 10, then the area of ∆ABC is
116. In triangle ABC shown in the adjoining figure, M is the mid-point of side BC, AB = 12 and AC = 16.
Points E and F are taken on AC and AB, respectively, and lines EF and AM intersect at G. If AE = 2AF
and EG/GF is X. Find the value of 10X.

117. Vertex E of equilateral triangle ABE is in the interior of square ABCD, and F is the point of
intersection of diagonal BD and line segment AE. If length AB is 1 + 3 and the area of ∆ABF is
given by A. Find (4A)2.

118. A ray of light originates from point A and travels in a plane, being reflected n times between lines AD
and CD, before striking a point B (which may be on AD or CD) perpendicularly and retracing its path
to A. (At each point of reflection the light makes two equal angles as indicated in the adjoining figure.
The figure shows the light path for n = 3.) If ∠CDA = 8°, what is the largest value n can have?

119. In a triangle with side of lengths a, b and c, (a + b + c) (a + b – c) = 3ab. The measure of the angle
opposite the side of length c is
120. A paper triangle with sides of lengths 3, 4, and 5 inches, as shown, is folded so that point A falls on
point B. Then the length in inches of the crease is L. Find the value of 8L.

H.O. 92, Rajeev Gandhi Nagar, Kota (Raj.) Mob. 97831-97831, 70732-22177, Ph. 0744-2423333 www.nucleuseducation.in 13

13 13
PCCP
121. Right triangle ABC has leg lengths AB = 20 and BC = 2l. Including AB and BC , how many line
segments with integer length can be drawn from vertex B to a point on hypotenuse AC ?
122. Farmer Pythagoras has a field in the shape of a right triangle. The right triangle's legs have lengths 3
and 4 units. In the corner where those sides meet at a right angle, he leaves a small unplanted square S
so that from the air it looks like the right angle symbol. The rest of the fiels is planted. The shortest
distance from S to the hypotenuse E is 2 units. Let the fraction of the field planted can be written has - .
Find the value of 10(y – x). Here x, y are co-prime numbers.

123. Triangle ABC with AB = 50 and AC = 10 has area 120. Let D be the midpoint of AB , and let E be the
midpoint of AC . The angle bisector of ∠BAC intersects DE and BC at F and G, respectively. What is
the area of quadrilateral FDBG?
124. Line segment AB is a diameter of a circle with AB = 24. Point C, not equal to A or B, lies on the circle. As
point C moves around the circle, the centroid (center of mass) of ∆ABC traces out a closed curve missing
two points. To the nearest positive integer, what is the area of the region bounded by this curve?
125. A square with side length x is inscribed in a right triangle with sides of length 3, 4, and 5 so that one
vertex of the square coincides with the right-angle vertex of the triangle. A square with side length y is
inscribed in another right angle triangle with sides of length 3, 4 and 5 so that one side of the square lies
on the hypotenuse of the triangle. Find the value of (7x + 37y).
126. Rectangle ABCD has AB = 3 and BC = 4. Point E is the foot of the perpendicular from B to diagonal
AC . If the area of ∆ADE is given by P. Find 25P.
127. Let ABC be an equilateral triangle. Extend side AB beyond B to a point B' so that BB' = 3AB. Similarly,
extend side BC beyond C to a point C' so that CC' = 3BC, and extend side CA beyond A to a point A' so
that AA' = 3CA. If the ratio of the area of ∆A'B'C' to the area of ∆ABC is x : 1. Find the value of x.
128. A thin piece of wood of uniform density in the shape of an equilateral triangle with side length 3 inches
weighs - 12 gm. A second piece of the same type of wood, with the same thickness, also in the shape of
an equilateral triangle, has side length of 5 inches. If W is the weight, in gm, of the second piece. Then
find [W], where [x] denotes the greatest integer less than or equal to x.
129. The isosceles right triangle ABC has right angle at C and area 12.5. The rays trisecting ∠ACB intersect
a(2 − b )
AB at D and E. If the area of ∆CDE can be written as . Find the product of a and b.
2

ANSWER KEY
Que . 1 2 3 4 5 6 7 8 9 10 11 12 13 14
Ans. 3 4 2 1 2 4 3 1 2 1 1 8 1 1
15. 45° 16. 0 17. 70 18. 30º 19. 30 20. AC = 3 5 , AB = 13 , BC = 2 13 21. 4: 1
26. 20 27. 48 28. 54 29. 33 30. 15 31. 01 32. 12 33. 87 34. 03 35. 12
36. 26 37. 30 38. 20 39. 01 40. 03 41. 06 42. 35 43. 20 44. 02 45. 50
46. 15 47. 02 48. 70 49. 77 50. 52 51. 12 52. 03 53. 20 54. 18 55. 14
56. 02 57. 75 58. 12 59. 80 60. 37 61. 42 62. 48 63. 35 64. 14 65. 56
66. 34 67. 36 68. 21 69. 15 70. 30 71. 10 72. 48 73. 00 74. 08 75. 12
76. 04 77. 27 78. 02 79. 33 80. 05 81. 01 82. 24 83. 21 84. 27 85. 90
86. 02 87. 03 88. 82 89. 03 90. 15 91. 01 92. 12 93. 56 94. 24 95. 48
96. 02 97. 43 98. 18 99. 03 100. 54 101. 04 102. 65 103. 27 104. 12 105. 05
106. 09 107. 26 108. 30 109. 64 110. 07 111. 10 112. 05 113. 03 114. 36 115. 40
116. 15 117. 12 118. 10 119. 60 120. 15 121. 13 122. 20 123. 75 124. 50 125. 72
126. 54 127. 37 128. 33 129. 75

H.O. 92, Rajeev Gandhi Nagar, Kota (Raj.) Mob. 97831-97831, 70732-22177, Ph. 0744-2423333 www.nucleuseducation.in 14

14 14
Rourkela Workshop
PRMO
Daily Practice Problems
Sub. : Mathematics DPP : 13

GEOMETRY
1. A circle is inscribed in a triangle ABC. It touches AB at D. AD= 5 cm, DB=3 cm and ∠A= 60°. Then BC is
(1) 8 cm (2) 13 cm (3) 20 cm (4) 31 cm
2. ABC is a triangle AB = 6cm, BC = 9cm, CA = 7cm. Circles are drawn with centres at A, B, C. The
circles with centres A and B touch externally. The circle with centre C touches these two circles
internally. The sum of the radii of these circles (in cm) is
(1) 22 (2) 17 (3) 44 (4) 111
3. In rectangle ABCD, AB = 2BC = 4 cm E and F are midpoints of AB and CD respectively. ESD and
ETC are arcs of circles centred at A and B respectively. If the perpendicular bisector line  of EF cuts
the arcs at S and T as in the diagram, then ST is equal to (in cm)

(1) (4 – 2 3 ) (2) (3 + 3 ) (3) (2 + 2 3 ) (4) (4 3 – 2)

4. An isosceles trapezoid is circumscribed about a circle of radius 2cm and the area of the trapezoid is
20cm2. The equal sides of the trapezoid have length equal to?
(1) 1 (2) 5 (3) 2 (4) 3
5. ABC and ADE are two secants of a circle of radius 3cm. A is at a distance of 5 cm from the centre of
the circle. The secants include an angle of 30°. The area of the ∆ACE is 10cm2. Then the area of the
∆ADB (in cm2) is____.
8 7 6 9
(1) (2) (3) (4)
5 5 5 5
6. In the figure below two equal circles S1, S2 of radii 2 units each touch each other. AB is the common
diameter. The tangent at B meets the tangent from A to the circle S2 at C as shown. If BC = k 2 then
the value of K is.

(1) 2 (2) 2.2 (3) 3 (4) 4


7. In the figure below, the triangle is a 3–4–5 sides triangle. Two equal circles are placed as in the figure.
The radius of each circle is
H.O. 92, Rajeev Gandhi Nagar, Kota (Raj.) Mob. 97831-97831, 70732-22177, Ph. 0744-2423333 www.nucleuseducation.in 1

1 1
PCCP

4 5 6 5
(1) (2) (3) (4)
5 8 11 7

8. In figure, PQ is a diameter of a circle with centre at O and OR ⊥ PQ, where R is a point on the circle. If
S is another point on the circle such that ∠RPS = 32°, then ∠QRS is

(1) 13° (2) 26° (3) 45° (4) None of these

9. In the given figure, AB is the diameter of a circle C (O, r). Chord CD is equal to radius OC. If AC and
BD intersect at P, find ∠APB.

10. In he given figure, ∠ BCP is equal to


(1) 60º (2) 100° (3) 50° (4)130º

11. In the figure given below, A and B are the centers of the two congruent circles with radius 17 units, If
AB = 30 units, the length of the common chord DC is :

(1) 25 units (2) 10 units (3) 10 units (4) 16 units


12. Two circles with centres A and B have radii 8 cm and 1 cm respectively. The distance AB is 13 cm. A third
circle with centre C and radius r cm touches both circles externally. If angle ACB is 90°, then value of r is:
(1) 11 (2) 4 (3) 5 (4) 2
13. Two circles of radii 2 and 3 cm touch each other externally. The length of direct common tangent to the
two circle will be :
(1) 2 6 cm (2) 26 cm (3) 5 cm (4) 2.4 cm
H.O. 92, Rajeev Gandhi Nagar, Kota (Raj.) Mob. 97831-97831, 70732-22177, Ph. 0744-2423333 www.nucleuseducation.in 2

2 2
PCCP
14. As shown in the figure, if O is the centre of the circle, then find the measure of the angle ∠BTC.

(1) 23º (2) 67º (3) 51º (4) 44°


15. In the following figure, AB = BC, AC = CD and ∠ACD = 90°. If the radius of the circle is "r" units
then find the length of the chord BC.

(1) r 2 − 1 (2) r 3 − 1 (3) r 2 − 2 (4) r 2 − 3


16. C1 and C3 are centres of two identical semi circles with radius 'r' units. The ratio of C1P1 to PQ is 1 : 7,
as shown in the figure. Find the radius of the circle inscribed in the region common to the identical
semicircles, given that the circle touches the two semicircles and the line segment P1P2.

7r 3r r 5r
(1) (2) (3) (4)
32 16 4 16
17. ABC is triangle with side lengths 13, 14 and 15 units. If I is the incentre and R is its circum radius, then
AI × BI × CI
the value of is equal to (in Sq. units)
R
(1) 130 (2) 64 (3) 126 (4) 100

18. Two adjacent vertices of a square are on a circle. The length of the side of the square is
4R 6R 8R 3R
(1) (2) (3) (4)
3 5 5 2
19. A circle with diameter AC is intersected by a secant at point B and D. The secant and the diameter
intersect at point P outside the circle. Perpendiculars AE and CF are drawn from the extremities of the
diameter to the secant. If EB = 2, and BD = 6, find DF.

(1) 2 (2) 3 (3) 4 (4) 5

20. In ∆ABC, in which AB = 12, BC = 18, and AC = 25, a semicircle is drawn so that diameter lies on AC ,
and so that it is tangent to AB and BC . If O is the center of the circle, find the measure of AO .

(1) 10 (2) 20 (3) 30 (4) 40


H.O. 92, Rajeev Gandhi Nagar, Kota (Raj.) Mob. 97831-97831, 70732-22177, Ph. 0744-2423333 www.nucleuseducation.in 3

3 3
PCCP
21. Triangle ABC is inscribed in a circle with diameter AD . A tangent to the circle at D cuts AB extended
at E and AC extended at F. If AB = 4, AC = 6, and BE= 8, find CF.

(1) 1 (2) 2 (3)4 (4) 5

22. A circle with radius 3 is inscribed in a square. Find the radius of the circle that is inscribed between two
sides of the square and the original circle.

(l) 3(3 + 2 2 ) (2) 3(3 – 2 2 ) (3) 3(2 + 2 2 ) (4) 3(2 – 2 2 )

23. AB is diameter of circle O. Two circles are drawn with AO and OB as diameters. In the region
between the circumferences, a circle D is inscribed tangent to the three previous circles. If the measure
of the radius of circle D is 8, find AB.

(1) 48 (2) 50 (3) 52 (4) 54

24. Find the ratio between the area of a square inscribed in a circle and an equilateral triangle circumscribed
about the same circle.

2 9 2 3 2 5 2 6
(1) (2) (3) (4)
9 9 9 9

25. The diagram shows a segment of a circle such that CD is perpendicular bisector of the chord AB. Given
that AB = 16 and CD = 4, find the diameter of the Circle.

26. n dots are drawn on the circumference of a circle. By joining all the dots to one another by
straight lines, the maximum number of regions that can be formed in the circle is counted. For example,
when n = 4 the maximum number of regions is 8.
H.O. 92, Rajeev Gandhi Nagar, Kota (Raj.) Mob. 97831-97831, 70732-22177, Ph. 0744-2423333 www.nucleuseducation.in 4

4 4
PCCP

What is the maximum number of regions that can be formed when n = 7?


27. In the diagram, P. Q and Rare three points on the circle whose centre is O. The lines PO and QR are
produced to meet at S. Suppose that RS = OP, and ∠PSQ = 12° and ∠POQ = Xº. Find the value of x.

28. ∆ABC is a triangle such that ∠C = 90°. Suppose AC = 156 cm, AB = 169 cm and the perpendicular
distance from C to AB is x cm. Find the value of x.
29. Circles C1 and C2 have radii 3 and 7 respectively. The circles intersect at distinct points A and B. A
point P outside C2 lies on the line determined by A and B at a distance of 5 from the center of C1. Point
Q is chosen on C2 so that PQ is tangent to C2 at Q. Find the length of the segment PQ.
30. Let C be a circle with radius 2006. Suppose n points are placed inside the circle and the distance
between any two points exceed 2006. What is the largest possible n?
31. In the following figure, AB is the diameter of a circle with centre at O. It is given that AB = 4 cm, BC =
3 cm, ∠ABD = ∠DBE. Suppose the area of the quadrilateral ABCD is x cm2 and the area of ∆DCE is y
x
cm2. Find the X value of the ratio .
y

32. Points A, B, C lie on a circle centered at O with radius 7. The perpendicular bisector of AB meets the
segment BC at P and the extension of AC at Q. Determine the value of OP · OQ.

H.O. 92, Rajeev Gandhi Nagar, Kota (Raj.) Mob. 97831-97831, 70732-22177, Ph. 0744-2423333 www.nucleuseducation.in 5

5 5
PCCP
33. Circles A, B and C are externally tangent to each other and internally tangent to circle D. Circles B and
a
C are congruent. Circle A has radius 1 and passes through the center of D. If the radius of circle B is
b
then the sum of a and b is?

34. Three semicircles of radius 1 are constructed on diameter AB of a semicircle of radius 2. The centers of
the small semicircles divide AB into four line segments of equal length, as shown. If the area of the
7π − a 3
shaded region that lies within the large semicircle but outside the smaller semicircles is , then
6
value of a = ?

35. In ∆ABC with side lengths AB = 13, AC = 12, and BC = 5, let O and I denote the circumcenter and
incenter, respectively. A circle with center M is tangent to the legs AC and BC and to the circumcircle
p
of ∆ABC. The area p of ∆MOI can be written as in simplest form. What is p2 + q2?
q
36. In the figure below, semicircles with centers at A and Band with radii 2 and 1, respectively, are drawn
in the interior of, and sharing bases with, a semicircle with diameter JK . The two smaller semicircles
are externally tangent to each other and internally tangent to the largest semicircle. A circle centered at
P is drawn externally tangent to the two smaller semicircles and internally tangent to the largest
a
semicircle. Radius of the circle centered at P is where a and b are coprime. What is 5a + 6b?
b

37. The diameter AB of a circle of radius 2 is extended to a point D outside the circle so that BD = 3. Point E is
chosen so that ED = 5 and the line ED is perpendicular to the line AD. Segment AE intersects the circle at
P
point C between A and E. The area of ∆ABC is where P and S are co-prime. What is P – 2S?
S
38. Circles with centers P, Q and R, having radii 1, 2 and 3, respectively, lie on the same side of line  and are
tangent to  at P', Q' and R', respectively, with Q' between P' and R'. The circle with centre Q is externally
tangent to each of the other two circles. The area of triangle PQR is a – b . What is a2 + b2?
39. Quadrilateral ABCD is inscribed inside a circle with ∠BAC = 70°, ∠ADB = 40°, AD= 4, and BC = 6.
What is (AC)2?
40. In ∆ΑΒC, AB = 86, and AC = 97. A circle with center A and radius AB intersects BC at points B and
X. Moreover BX and CX have integer lengths. What is BC?
H.O. 92, Rajeev Gandhi Nagar, Kota (Raj.) Mob. 97831-97831, 70732-22177, Ph. 0744-2423333 www.nucleuseducation.in 6

6 6
PCCP
41. Circle C1 has its center O lying on circle C2. The two circles meet at X and Y. Point Z in the exterior of
C1 lies on circle C2 and XZ = 13, OZ = 11, and YZ = 7. The radius of circle C1 is R . What is R?
42. Let AB be a diameter of a circle and C be a point on AB with 2.AC = BC. Let D and E be points on
the circle such that DC ⊥ AB and DE is a second diameter. The ratio of the area of ∆DCE to the area
21
of ∆ΑΒD is . What is y?
y

43. The diagram shows a sector OAB of a circle, centre O and radius 8 cm, in which ∠AOB = 120°.
Another circle or radius r cm is to be drawn through the points O, A and B. Find the value of r.

44. The figure below shows two circles with centres A and B, and a line L which is a tangent to the circles
at X and Y. Suppose that XY = 40 cm, AB = 41 cm and the area of the quadrilateral ABYX is 300 cm2.
b
If a and b denote the areas of the circles with centre A and centre B respectively, find the value of .
a

45. 
Let A1, A2, A3, A4, A5 and A6 be six points on a circle in this order such that A  
1A 2 = A 2 A 3 , A 3 A 4 =
   
A 4 A 5 and A 5 A 6 = A 6 A1 , where A1A 2 denotes the arc length of the arc A1A2 etc. It is also known
that ∠A1A3A5 = 72º. Find the size of ∠A4A6A2 in degrees.

46. The figure below shows a circle with diameter AB. C and D are points on the circle on the same side of
AB such that BD bisects ∠CBA. The chords AC and BD intersect at E. It is given that AE = 169 cm
and EC = 119 cm. If ED = x cm, find the value of x.

H.O. 92, Rajeev Gandhi Nagar, Kota (Raj.) Mob. 97831-97831, 70732-22177, Ph. 0744-2423333 www.nucleuseducation.in 7

7 7
PCCP

47. Two circles C1 and C2 of radii 10 cm and 8 cm respectively are tangent to each other internally at a
point A. AD is the diameter of C1 and P and M are points on C1 and C2 respectively such that PM is
tangent to C2 , as shown in the figure below. If PM = 20 cm and ∠PAD = Xº, find the value of x.

48. Let CD be a chord of a circle Γ1 and AB a diameter of Γ1, perpendicular to CD at N with AN > NB. A
circle Γ2 centred at C with radius CN intersects Γ1 at points P and Q, and the segments PQ and CD
intersect at M. Given that the radii of Γ1 and Γ2 are 61 and 60 respectively, find the length of AM.
49. Let B be a point on the circle centred at O with diameter AC and let D and E be the circumcentres of
4
the triangles OAB and OBC respectively. Given that sin ∠BOC = and AC = 24, find the area of the
5
triangle BDE.
50. Circle A has radius 100. Circle B has an integer radius r < 100 and remains internally tangent to circle
A as it rolls once around the circumference of circle A. The two circles have the same points of
tangency at the beginning and end of circle B's trip. How many possible values can r have?
51. Many cathedrals have windows with portions containing a ring of congruent circles that are
circumscribed by a larger circle, In the figure shown, the number of smaller circles is four. If the ratio
of the sum of the areas of the four smaller circles to the area of the larger circle is 2[x + 4y 2 ] then
find the value of x2 + y3

52. Points A and B lie on a circle centered at O, and ∠AOB = 60°. A second circle is internally tangent to
the first and tangent to both OA and OB . If the ration of the area of the smaller circle to that of the
x
larger circle is then find the value of x2 + y2.
y
53. Points A and Bare on a circle of radius 5 and AB = 6. Point C is the midpoint of the minor arc AB.
What is the length of the line segment AC2?

54. Four circles of radius 1 are each tangent to two sides of a square and externally tangent to a circle of
5x
radius 2, as shown. If the area of the square is x + y 2 then find the value of ?
x−y

H.O. 92, Rajeev Gandhi Nagar, Kota (Raj.) Mob. 97831-97831, 70732-22177, Ph. 0744-2423333 www.nucleuseducation.in 8

8 8
PCCP

55. Circles centered at A and B each have radius 2, as shown. Point O is the midpoint of AB , and OA = 2
2 . Segments OC and OD are tangent to the circles centered at A and B, respectively, and EF is a
common tangent. What is the area of the shaded region ECODF to the nearest integer? (Take π = 3 .14)

56. A circle of radius 1 is surrounded by 4 circles of radius r as shown. Find the value of r to the nearest
whole number.

57. A circle of radius 1 is tangent to a circle of radius 2. The sides of ∆ABC are tangent to the circles as
shown, and the sides AB and AC are congruent. What is the area of ∆ABC to the nearest integer?

58. Circles with centers A and B have radii 3 and 8, respectively. A common internal tangent intersect at C
and D, respectively. Lines AB and CD intersect at E, and AE = 5. What is 3CD?

H.O. 92, Rajeev Gandhi Nagar, Kota (Raj.) Mob. 97831-97831, 70732-22177, Ph. 0744-2423333 www.nucleuseducation.in 9

9 9
PCCP

59. Circles with centers O and P have radii 2 and 4, respectively, and are externally tangent. Points A and B are
on the circle centered at O, and points C and D are on the circle centered at P, such that AD and BC are
common external tangents to the circles. If the area of hexagon AOBCPD is x 3 then find the value of x?

60. Let AB be a diameter of a circle and C be a point or, AB with 2 · AC = BC. Let D and E be points on
the circle such that DC ⊥ AB and DE is a second diameter. If is the ratio of the area of ∆DCE to the
x 1
area of ∆ΑΒD is then find +y
y x

61. Two distinct lines pass through the centre of three concentric circles of radii 3, 2, and 1. The area of the
shaded region in the diagram is 8/13 of the area of the unshaded region. Find 3 times the measure of the
acute angle formed by the two lines? (Note: π radians is 180 degrees.)

62. Circles A, B, and C are externally tangent to each other and internally tangent to circle D. Circles B and
C are congruent. Circle A has radius 1 and passes through the centre of D. What is three times the
square root of diameter of circle B?

H.O. 92, Rajeev Gandhi Nagar, Kota (Raj.) Mob. 97831-97831, 70732-22177, Ph. 0744-2423333 www.nucleuseducation.in 10

10 10
PCCP
63. In ∆ABC we have, AB = 7, AC = 8, and BC = 9. Point D is on the circumscribed circle of the triangle
 3AD 
so that AD bisects ∠BAC. What is the value of  ?
 CD 
64. A circle of radius 1 is internally tangent to two circles of radius 2 at points A and B, where AB is a
diameter of the smaller circle. If the area of the region, shaded in the figure, that is outside the smaller
 aπ 
circle and inside each of the two larger circles is represented by  + b 3  , then find value of (a + b).
 3 

65. A semicircle of diameter 1 sits at the top of a semicircle of diameter 2, as shown. The shaded area
inside the smaller semicircle and outside the larger semicircle is called a lune. If the area of this lune is
 3 π π
in the form of  −  then find   .
 x y 
 y

66. Three semicircles of radius 1 are constructed on diameter AB of a semicircle of radius 2. The centers
of the small semicircles divide AB into four line segments of equal length, as shown. Then the area of
shaded region that lies within the large semicircle but outside the smaller semicircles is given by
a c
 b π − d  . Find the value of (a + b + c + d ).
2 2 2 2

 

H.O. 92, Rajeev Gandhi Nagar, Kota (Raj.) Mob. 97831-97831, 70732-22177, Ph. 0744-2423333 www.nucleuseducation.in 11

11 11
PCCP
67.  has center C and radius 1. Point D is on AB
Semicircle AB  and CD ^ AB . Extend BD  and AD  to E
 and BF
and F respectively, so that circular arcs AE  have B and A as their respective centers circular
arc EF has center D. The area of the shaded "smile", AEFBDA is (2π – π 2 – k), find k.

68. A circle of radius r has chord AB length 10 and CD of length 7. When AB and CD are extended
through B and C, respectively, they intersect at P, which is outside the circle. If ∠APD = 60° and BP =
8, then r2 =
69. A circle with centre O is tangent to the coordinate axes and to the hypotenuse of the 30° – 60° – 90° triangle
ABC as shown, where AB = 1. What is the radius of the circle (round off to nearest natural number)?

70. A circle is circumscribed about a triangle with sides 3, 4, and 5, thus dividing the interior of the circle
into four regions. Let A, B, and C be the areas of the non-triangular regions, with C being the largest.
Then – (A + B – C) = ?
71. If circular arcs AC and BC have centers at B and A, respectively, then there exists a circle tangent to
 and BC
both AC  , and to AB . If the length of BC
 is 12, then the circumference of the circle is

72. In the adjoining figure, circle K has diameter AB; circle L is tangent to circle K and to AB at the center
of circle K; and circle M is tangent to circle K, to circle L and to AB. The ra tio of the area of circle K
to the area of circle M is

73. A circle with area A1 is contained in the interior of a larger circle with area A1 + A2. If the radius of
the larger circle is 3, and if A1, A2, A1 + A2 is an arithmetic progression, and the radius of the smaller
circle is given by r. Find r2.

H.O. 92, Rajeev Gandhi Nagar, Kota (Raj.) Mob. 97831-97831, 70732-22177, Ph. 0744-2423333 www.nucleuseducation.in 12

12 12
PCCP
74. Two circles of radius 5 are externally tangent to each other and are internally tangent to a circle of
m
radius 13 at m points A and B, as shown in the diagram. The distance AB can be written in the form
n
, where m and n are relatively prime positive integers. What is m + n?

75. In the figure below, N congruent semicircles are drawn along a diameter of a large semicircle, with
their diameters covering the diameter of the large semicircle with no overlap. Let A be the combined
area of the small semicircles and B be the area of the region inside the large semicircle but outside the
small semicircles. The ratio A : B is 1 : 18. What is N?

76. Seven cookies of radius 1 inch are cut from a circle of cookie dough, as shown. Neighboring cookies are
tangent, and all except the center cookie are tangent to the edge of the dough. The leftover scrap is reshaped
to form another cookie of the same thickness. If the radius in inches of the scrap cookie is R. Find 4R2.

77. Six points are equally spaced around a circle of radius 1. Three of these points are the vertices of a
4A
triangle that is neither equilateral nor isosceles. If the area of this triangle is A, find .
3
78. Mary divides a circle into 12 sectors. The central angles of these sectors, measured in degrees, are all
integers and they form an arithmetic sequence. What is the degree measure of the smallest possible
sector angle?
79. Externally tangent circles with centers at points A and B have radii of lengths 5 and 3, respectively.
A line externally tangent to both circles intersects ray AB at point C. What is BC?
80. Three circles with radius 2 are mutually tangent. If the total area of the circles and the region bounded
by them, as shown in the figure is 10π + 4 k , find k.

81. Circles A, B, and C each have radius 1. Circles A and B share one point of tangency. Circle C has a point of
tangency with the midpoint of AB . What is the area inside Circle C but outside circle A and cirde B?

H.O. 92, Rajeev Gandhi Nagar, Kota (Raj.) Mob. 97831-97831, 70732-22177, Ph. 0744-2423333 www.nucleuseducation.in 13

13 13
PCCP

82. In the given circle, the diameter EB is parallel to DC , and AB is parallel to ED . The angles AEB and
k
ABE are in the ratio 4 : 5. If the degree measure of angle BCD is k, find .
5

83. A circle with center O has area 156π. Triangle ABC is equilateral, BC is a chord on the circle, OA = 4
3 , and point O is outside ∆ΑΒC. What is the side length of ∆ΑΒC?
84. In the figure below, ABC is an isosceles triangle inscribed in a circle with centre O and diameter AD,
with AB = AC. AD intersects BC at E, and F is the midpoint of OE. Given that BD is parallel to FC and
BC = 2 5 cm, find square of the length CD (in cm).

Answer Key
Que. 1 2 3 4 5 6 7 8 9 10 11 12 13 14 15 16 17 18 19 20
Ans. 2 2 1 2 1 1 4 1 3 4 4 2 1 4 3 1 2 3 1 1
Que. 21 22 23 24
Ans. 2 2 1 2
25. 20 26. 57 27. 36 28. 60 29. 04 30. 05 31. 07
32. 49 33. 17 34. 03 35. 53 36. 72 37. 66 38. 40
39. 36 40. 61 41. 30 42. 63 43. 08 44. 16 45. 54
46. 65 47. 60 48. 78 49. 45 50. 8 51. 35 52. 82
53. 10 54. 11 55. 04 56. 02 57. 23 58. 44 59. 24
60. 02 61. 60 62. 04 63. 05 64. 03 65. 06 66. 92
67. 01 68. 73 69. 02 70. 06 71. 27 72. 16 73. 03
74. 69 75. 19 76. 08 77. 02 78. 08 79. 12 80. 03
81. 02 82. 26 83. 06 84. 06
H.O. 92, Rajeev Gandhi Nagar, Kota (Raj.) Mob. 97831-97831, 70732-22177, Ph. 0744-2423333 www.nucleuseducation.in 14

14 14
Rourkela Workshop
PRMO
Daily Practice Problems
Sub. : Mathematics DPP : 14

GEOMETRY
1. ABCD is a square and K and ∠ are points on BC and DC. AM is perpendicular to ∠K. ∠AKM =
∠AKB. Find the measure of ∠LA.K

(1) 45° (2) 70° (3) 55° (4) 35°

2. PQRS is a parallelogram. MP and NP divide ∠SPQ into three equal parts (∠MPQ > ∠NPQ) and MQ
and NQ divide ∠RQP into 3 equal parts (∠MQP > ∠NQP). If k(∠PNQ = ∠PMQ) then k =
1 2 1
(1) (2) 1 (3) (4)
2 3 3

3. ABCD is a trapezium with AB || CD and AB < DC. AC and BD intersect at E, EF || AB, intersecting
BC at F. Given that AB = 20, CD= 80, BC = 100, then EF is
(1) 10 (2) 12 (3) 16 (4) 18

4. In the adjoining figure ABCD is a parallelogram of perimeter 21. It is subdivided into smaller
parallelograms by 7 drawing Lines parallel to the sides. The numbers shown are the respective
perimeters of the parallelograms in which they are marked. (For example the perimeter of the
parallelogram LMNP is 11). Find the perimeter of the shaded parallelogram.

(1) 7 (2) 8 (3) 10 (4) 11

5. A polygonal prism has 2010 edges. The number of the faces of the prism will be
(1) 772 (2) 670 (3) 1005 (4) 672

6. The diagonals of a convex quadrillateral are perpendicular. If AB = 4, AD= 5, CD= 6, then length of
BC is _____.
(1) 2 3 (2) 4 3 (3) 3 3 (4) 5 3

7. The quadrilateral ABCD is inscribed in a circle. The diagonals AC and BD cut at Q. The sides DA and
CB are produced to meet at P. If CD = CP = DQ then the measure of ∠CAD is
H.O. 92, Rajeev Gandhi Nagar, Kota (Raj.) Mob. 97831-97831, 70732-22177, Ph. 0744-2423333 www.nucleuseducation.in 1

1 1
PCCP
(1) 45° (2) 70° (3) 60° (4) 55°

8. ABCD is a convex quadrilateral in which AD = 3 , ∠A = 60°, ∠D = 120° and AB + CD = 2AD. M is


the midpoint of BC. Then DM =
3 3 2 2
(1) (2) (3) (4)
2 2 3 3
9. In a rectangle ABCD where AB = 6, BC = 3, point P is chosen on AB such that ∠APD = 2∠CPB. Then
AP =
(1) (1 – 7 ) (2) (4 – 7 ) (3) (3 – 7 ) (4) (5 – 7 )
AP 1
10. ABCD is a parallelogram P is a point on AD such that = . Q is the point of intersection of
AD 2013
AQ
AC and BP. Calculate = _____.
AC
1 2 3 4
(1) (2) (3) (4)
2014 2014 2014 2014

11. ABCD is a square. With centres B and C and radius equal to the side of the square, circles are drawn to
cut one another at E inside the square. ∠BDE is equal to
1
(1) 22 º (2) 30° (3) 15° (4) 37º
2

12. ABCD is a rectangle in which AB = 8, AD= 9. E is on AD such that DE = 4. H is on BC such that BH


= 6. EC and AH cut at G. GF is drawn perpendicular to AD produced. Then GF =
(1) 20 (2) 22 (3) 18 (4) 15

13. In the right trianlge ABC, ∠C = 90°, BC = 12 cm, AC = 6 cm, the perpendicular bisector of AB
intersects AB and BC at D and E respectively. Find CE.
(1) 4.5 (2) 3.5 (3) 2.5 (4) 1.5

1
14. In the rectangle ABCD, CE ⊥ DB at E, BE = BD and CE = 5 cm. Find the length of AC.
4
20 3 25 3 26 3 29 3
(1) (2) (3) (4)
3 3 3 3

15. ABCD is a rectangle with AD = 2; AB = 4. P is on AB such that AP : PB = 2 : 1, CE ⊥ DP at E. Find CE.


16. In the figure below, if
∠A + ∠B + ∠C + ∠D + ∠E + ∠F + ∠G = 10x degrees,
then what is x?

H.O. 92, Rajeev Gandhi Nagar, Kota (Raj.) Mob. 97831-97831, 70732-22177, Ph. 0744-2423333 www.nucleuseducation.in 2

2 2
PCCP

17. In the diagram, AB | | EF | | DC. Given that AC + BD = 250, BC = 100 and EC + ED = 150 find CF.

18. In the following figure, AD = AB, ∠DAB = ∠DCB = ∠AEC = 90° and AE = 5. Find the area of the
quadrangle ABCD.

19. A pentagon ABCDE is inscribed in a circle of radius 10 such that BC is parallel to AD and AD
intersects CE at M. The tangents to this circle at B and E meet the extension of DA at a common point
P. Suppose PB = PE = 24 and ∠BPD =30°. Find BM.
20. Consider the following "star" figure.

Given that ∠p + ∠q + ∠r + ∠s + ∠t = 500° and ∠A+ ∠B + ∠C + ∠D + ∠E = Xº, find the value of 180 – x.
21. Let ABCDEF be a hexagon such that the diagonals AD, BE and CF intersect at the point O, and the
area of the triangle formed by any three adjacent points is 2 (for example, area of ∆BCD is 2). Find the
area of the hexagon.
22. In the diagram below, ABCD is a square. The points A, B and G are collinear. The line segments AC
and DG intersect at E and the line segments DG and BC intersect at F. Suppose that DE = 15 cm, EF =
9 cm, and FG = x cm. Find the value of x.

H.O. 92, Rajeev Gandhi Nagar, Kota (Raj.) Mob. 97831-97831, 70732-22177, Ph. 0744-2423333 www.nucleuseducation.in 3

3 3
PCCP

23. Let P be a 30-sided polygon inscribed in a circle. There are N number of triangles whose vertices are
the vertices of P such that any two vertices of each triangle are separated by at least three other vertices
N
of P. Then determine value of .
100
24. The diagram shows two identical squares, ABCD and PQRS, overlapping each other in such a way that
their edges are parallel, and a circle of radius (2 – 2 ) cm covered within these squares. Find
the length of the square ABCD.

25. In the following figure, AB | | DC, AB = b, CD = a and a < b. Let S be the area of the trapezium ABCD.
Suppose the area of ∆BOC is 2S/9. Find the value of b/a.

26. W, X, Y and Z are the midpoints of the four sides of parallelogram ABCD. P is a point on the line
segment YZ. What percent of the area of parallelogram ABCD is triangle PXW?

27. Four rods are connected together with flexible joints at their ends to make a quadrilateral as shown.
Rods PQ = 40 cm, RS = 20 cm, PS = 60 cm and ∠QPS = ∠RSP = 60°. Find the value of x where xº =
180° – ∠QRS

H.O. 92, Rajeev Gandhi Nagar, Kota (Raj.) Mob. 97831-97831, 70732-22177, Ph. 0744-2423333 www.nucleuseducation.in 4

4 4
PCCP
28. The diagram below shows a quadrilateral ABCD where AB = 10, BC = 6, CD = 8 and DA = 2. The
diagonals AC and BD intersect at the point O and that ∠COB = 45°. Find the area of the quadrilateral
ABCD.

29. Points K, L, M, and N lie in the plane of the square ABCD so that AKB, BLC, CMD, and DNA are
equilateral triangles. If ABCD has an area of 16, If the area of KLMN is a + b 3 then find b.

30. Square ABCD has sides of length 4, and M is the midpoint of CD . A circle with radius 2 and center M
a
intersects a circle with radius 4 and center A at points P and D. If is the distance from P to AD is
b
then a + b = ?

31. Let ABCD be a rhombus with AC = 16 and BD = 30. Let N be a point on AB , and let P and Q be the
feet of the perpendiculars from N to AC and BD , respectively. Which of the following is closest to the
minimum possible value of PQ?

32. In rectangle ABCD, points F and G lie on AB so that AF = FG = GB and E is the midpoint of DC . Also,
AC intersects EF at H and EG at J. The area of the rectangle ABCD is 70. Find the area of triangle EHJ.
33. Square ABCD has side Length 30. Point P lies inside the square so that AP = 12 and BP = 26. The
centroids of ∆ABP, ∆BCP, ∆CDP and ∆DAP are the vertices of a convex quadrilateral. The area of
A
that quadrilateral is A. What is ?
4

H.O. 92, Rajeev Gandhi Nagar, Kota (Raj.) Mob. 97831-97831, 70732-22177, Ph. 0744-2423333 www.nucleuseducation.in 5

5 5
PCCP

34. The ratio of the short side of a certain rectangle to the long side is equal to the ratio of the long side to
the diagonal. The square of the ratio of the short side to the long side of this rectangle is in the form of
a −b
. What is (a + b – c)2?
c
35. The area of the shaded region of the given 8 × 5 rectangle is an improper fraction with co-prime digits.
What is the sum of numrator and denominator?

36. The five small shaded squares inside this unit square are congruent and have disjoint interiors. The midpoint
of each side of the middle square coincides with one of the vertices of the other four small squares a as
a− 2
shown. The common side length is , where a and b are positive integers. What is a + b ?
b

37. Let ABCD be a square. Let E, F, G and H be the centers, respectively, of equilateral triangles with
bases AB , BC , CD and DA , each exterior to the square. The ratio of the area of square EFGH to the
P− Q
area of square ABCD is . Find (P3 + Q2 + R).
R
38. For some positive integers p, there is a quadrilateral ABCD with positive integer side lengths, perimeter
p, right angles at B and C, AB = 2, and CD = AD. How many different values of p < 2015 possible?
39. In ∆ABC, ∠C = 90° and AB = 12. Squares ABXY and ACWZ are constructed outside of the triangle.
The points X, Y, Z and W lie on a circle. The perimeter of the triangle is in the form of a + b 2 . What
is (a + b)?
40. Convex quadrilateral ABCD has AB = 9 and CD = 12. Diagonals AC and BD intersect at E, AC = 14,
and ∆AED and ∆BEC have equal areas. What is 7 × AE?
41. Trapezoid ABCD has AD || BC, BD = 1, ∠DBA = 23°, and ∠BDC = 46°. The ratio BC : AD is 9 : 5.
p
CD = where gcd (p, q) = 1. What is p2 + q2?
q
H.O. 92, Rajeev Gandhi Nagar, Kota (Raj.) Mob. 97831-97831, 70732-22177, Ph. 0744-2423333 www.nucleuseducation.in 6

6 6
PCCP
2
42. Five identical rectangles of area 8 cm are arranged into a large rectangle as shown.

Find the perimeter of this large rectangle.


43. The lengths of the sides of a quadrilateral are 2006 cm, 2007 cm, 2008 cm and x cm. If x is an integer
and its largest possible value is given as 103a + b then find a + b.
44. Let ABCD be a quadrilateral inscribed in a circle with diameter AC, and let E be the foot of
perpendicular from D onto AB, as shown in the figure below. If AD = DC and the area of quadrilateral
ABCD is 24 cm2, find the square of Length of DE (in cm).

45. In the figure below, ABCD is a rectangle, E is the midpoint of AD and F is the midpoint of CE. If the
area of triangle BDF is 12 cm2, find the area of rectangle ABCD in cm2.

46. Given that ABCD is a square. Points E and F lie on the side BC and CD respectively, such that BE =
1
CF = AB. G is the intersection of BF and DE. If
3
Area of ABGD m
=
Area of ABCD n
is in the lowest term, find the value of m + n.

47. Rectangle ABCD has AB = 4 and BC = 3. Segment EF is constructed through B so that EF is


perpendicular to DB, and A and C lie on DE and DF, respectively. What is EF to the nearest integer?
48. As shown below, convex pentagon ABCDE has sides AB = 3, BC = 4, CD = 6, DE = 3, and EA = 7.
The pentagon is originally positioned in the plane with vertex A at the origin and vertex B on the
positive x-axis. The pentagon is then rolled clockwise to the right along the x-axis. What is the length
side which will touch the point x = 2009 on the x-axis?

H.O. 92, Rajeev Gandhi Nagar, Kota (Raj.) Mob. 97831-97831, 70732-22177, Ph. 0744-2423333 www.nucleuseducation.in 7

7 7
PCCP

49. Rectangle ABCD has AB = 8 and BC = 6. Point M is the midpoint of diagonal AC , and E is on AB
with ME ⊥ AC . What is the area of ∆AME to the nearest integer?
50. Each of the sides of a square S1 with area 16 is bisected, and a smaller square S2 is constructed using the
bisection points as vertices. The same process is carried out on S2 to construct an even smaller square
S3. What is the area of S3?
51. Rectangle PQRS lies in a plane with PQ = RS = 2 and QR = SP = 6. The rectangle is rotated 90°
clockwise about R, then rotated 90° clockwise about the point S that moved to after the first rotation.
What is the length of the path traveled by point P to the nearest integer? (Take π = 3.14)
52. Trapezoid ABCD has bases AB and CD and diagonals intersecting at K. Suppose that AB = 9. DC =
12. and the area of ∆AKD is 24. What is the area of trapezoid ABCD?
53. Quadrilateral ABCD has AB = BC = CD, ∠ABC = 70° and ∠BCD = 170°. What is the degree measure
of ∠BAD?
54. The angles of quadrilateral ABCD satisfy ∠A = 2∠B = 3∠C = 4∠D. What is the degree measure
of ∠Α/2, rounded to the nearest whole number?
55. The 8 × 18 rectangle ABCD is cut into two congruent hexagons, as shown, in such a way that the two
hexagons can be repositioned without overlap to form a square. What is y?

56. In rectangle ADEH, points B and C trisect AD , and points G and F trisect HE . In addition, AH = AC
= 2. What is twice of the area of quadrilateral WXYZ shown in the figure?

57. What is the tens digit in the sum 7! + 8! + 9! + · · · + 2006!?


58. Rhombus ABCD is similar to rhombus BFDE. The area of rhombus ABCD is 24, and ∠BAD = 60°.
What is the area of rhombus BFDE?

59. In ABC we have AB = 25, BC = 39, and AC = 42. Points D and E are on AB and AC respectively, with
AD = 19 and AE = 14. What is the ratio of the area of triangle ADE to the area of quadrilateral BCED
m
is then find m + n.
n
H.O. 92, Rajeev Gandhi Nagar, Kota (Raj.) Mob. 97831-97831, 70732-22177, Ph. 0744-2423333 www.nucleuseducation.in 8

8 8
PCCP
60. In trapezoid ABCD we have AB parallel to DC , E as the midpoint of BC , and F as the midpoint of
DA . The area of ABEF is twice the area of FECD. What is AB/DC?
61. Points E and F are located on square ABCD so that ∆BEF is equilateral. What is the ratio of the area of
∆DEF to that of ∆ABE?

62. Square ABCD has side length 2. A semicircle with diameter AB is constructed inside the square, and
the tangent to the semicircle from C intersects side AD at E. Find to 2 CE .

63. In rectangle ABCD, we have AB = 8, BC = 9, H is on BC with BH = 6, E is on AD with DE = 4, line


EC intersects line AH at G, and F is on line AD with GF ⊥ AF . Find the length GF.

64. In rectangle ABCD, AB = 5 and BC = 3. Points F and G are on CD so that DF = 1 and GC = 2. Lines
AF and BG intersect at E. Then the area of ∆AEB is given by S. Find the value of 25.

H.O. 92, Rajeev Gandhi Nagar, Kota (Raj.) Mob. 97831-97831, 70732-22177, Ph. 0744-2423333 www.nucleuseducation.in 9

9 9
PCCP

65. In trapezoid ABCD with bases AB and CD, we have AB =52, BC = 12, CD= 39, and DA = 5. The area
of ABCD is given by 10A. Find the value of A.

66. A regular octagon ABCDEFGH has sides of length two. Then the area of ∆ADG is given by (a + b c ).
Find the product of a, b & c.

67. A regular octagon is formed by cutting an isosceles right triangle from each of the corners of a square
with sides of length 20. Let the length of each side of the octagon is given by a( b – c). Find the
product of a, b and c.
68. A right circular cylinder with its diameter equal to its height is inscribed in a right circular cone. The
cone has diameter 10 and altitude 12, and the axes of the cylinder and cone coincide. Then the radius of
the cylinder is R. Find the value of (11R).
69. In trapezoid ABCD, AB and CD are perpendicular to AD , with AB + CD = BC, AB < CD, and AD =
7. What is (4AB · CD)?
70. A square is cut into three rectangles along two lines parallel to a side, as shown. If the perimeter of each
of the three reactangles is 24, then the area of the original square is

71. In the figure, ABCD is an isosceles trapezoid with side lengths AD = BC = 5, AB = 4, and DC = 10.
The point C is on DF and B is the midpoint of hypotenuse DE in the right triangle DEF. Then CF =

72. In the figure, ABCD is a quadrilateral with right angles at A and C. Points E and F are on AC , and DE
1
and BF are perpendicular to AC . If AE = 3, DE = 5 and CE = 7, then the value of BF – is
5

H.O. 92, Rajeev Gandhi Nagar, Kota (Raj.) Mob. 97831-97831, 70732-22177, Ph. 0744-2423333 www.nucleuseducation.in 10

10 10
PCCP

73. If ABCD is a 2 × 2 square, E is the midpoint of AB , F is the midpoint of BC , AF and DE intersect at


k
I, and BD and AF intersect at H, and the area of quadrilateral BEIH is , find k.
2k + 1

74.  has center C and radius 1. Point D is on AB


Semicircle AB  and CD ^ AB . Extend BD  and AD  to E
 and BF
and F respectively, so that circular arcs AE  have B and A as their respective centers, circular
arc EF has center D. The area of the shaded “smile”, AEFBDA is (2π – π 2 – k), find k.

75. Triangle PAB and square ABCD are in perpendicular planes. Given that PA = 3, PB = 4, and AB = 5,
what is PD2?

76. In quadrilateral ABCD, it is given that ∠A= 120°, angles B and D are right angles, AB = 13, and AD =
46. Then AC =
77. In the adjoining figure ABCD is a square and CMN is an equilateral triangle. If the area of ABCD is one
square inch, and the area of CMN in square inches is given by (2 a – b). Find the product of a and b.

H.O. 92, Rajeev Gandhi Nagar, Kota (Raj.) Mob. 97831-97831, 70732-22177, Ph. 0744-2423333 www.nucleuseducation.in 11

11 11
PCCP
78. A three-dimensional rectangular box with dimensions X, Y and Z has faces whose surface areas are 24,
24, 48, 48, 72, and 72 square units. What is X + Y + Z?
79. A closed box with a square base is to be wrapped with a square sheet of wrapping paper. The box is centered
on the wrapping paper with the vertices of the base lying on the mid lines of the square sheet of paper, as
shown in the figure on the left. The four corners of the wrapping paper are to be folded up over the sides and
brought together to meet at the center of the top of the box, point A in the figure on the right. The box has
base length 2 meter and height 5 meter. What is the area of the sheet of wrapping paper?

80. Let ABCDEF be a regular hexagon with side length 1. Denote by X, Y and Ζ the midpoints of AB ,
CD , and EF , respectively. If the area of the convex hexagon whose interior is the intersection of the
a b
interiors of ∆ACE and ∆XYZ is given by . Find the product of a and b.
32
81. In rectangle ABCD, AB = 6 and BC = 3. Point E between B and C, and point F between E and C are
such that BE = EF = FC. Segments AE and AF intersect BD at P and Q, respectively. The ratio BP:
PQ : QD can be written as r : s : t, where the greatest common factor of r, s and t is 1. What is r + s + t ?
82. A quadrilateral is inscribed in a circle of radius 20 2 . Three of the sides of this quadrilateral have
length 20. What is the length of the fourth side?
83. Chetan decided to fence in his rectangular garden. He bought 20 fence posts, placed one on each of the
four comers, and spaced out the rest evenly along the edges of the garden, leaving exactly 4 yards
between neighboring posts. The longer side of his garden, including the corners, has twice as many
posts as the shorter side, including the corners. If the area, in square yards, of Carls garden is given by
A. Find [400 – A].
84. Rectangle ABCD has AB = 5 and BC = 4. Point E lies on AB so that EB = 1, point G lies on BC so
that CG = 1 and point F lies on CD so that DF = 2. Segments AG and AC intersect EF at Q and P,
 PQ 
respectively. What is the value of 911 − ?
 EF 

85. The ratio of the length to the width of a rectangle is 4: 3. If the rectangle has diagonal of length d, then
the area may be expressed as kd2 for some constant k. Find the value of 100k?

H.O. 92, Rajeev Gandhi Nagar, Kota (Raj.) Mob. 97831-97831, 70732-22177, Ph. 0744-2423333 www.nucleuseducation.in 12

12 12
PCCP
86. For some positive integers p, there is a quadrilateral ABCD with positive integer side Lengths, perimeter p,
right angles at Band C, AB = 2, and CD= AD. How many different values of p < 2015 are possible?
87. A rectangular box measures a × b × c, where a, b, and c are integers and 1 ≤ a ≤ b ≤ c. The volume and
surface area of the box are numerically equal. How many ordered triples (a, b, c) are possible?
88. A regular hexagon has side length 6. Congruent arcs with radius 3 are drawn with the center at each of the
vertices, creating circular sectors as shown. The region inside the hexagon but outside the sectors is shaded
as shown. The area of the shaded region can be written as (a 3 – πb). Then find the sum of a and b.

89. In rectangle ABCD, AB = 1, BC = 2, and points E, F, and G are midpoints of BC , CD , and AD ,


respectively. Point H is the midpoint of GE . What is the reciprocal of area of the shaded region?

90. In rectangle ABCD, DC = 2CB and points E and F lie on AB so that ED and FD trisect ∠ADC as shown.
If the ratio of the area of ∆DEF to the area of rectangle ABCD is given by A. Find the value of 10 3 A.

91. Trapezoid ABCD has parallel sides AB of length 33 and CD of length 21. The other two sides are of
lengths 10 and 14. The angles at A and B are acute. What is the length of the shorter diagonal of ABCD?
92. All diagonals are drawn in a regular octagon. At how many distinct points in the interior of the octagon
(not on the boundary) do two or more diagonals intersect?
93. A wire is cut into two pieces, one of length a and the other of length b. The piece of length a is bent to
form an equilateral triangle, and the piece of length b is bent to form a regular hexagon. The triangle
4a 2
and the hexagon have equal area. What is 2 ?
b

H.O. 92, Rajeev Gandhi Nagar, Kota (Raj.) Mob. 97831-97831, 70732-22177, Ph. 0744-2423333 www.nucleuseducation.in 13

13 13
PCCP
94 . Three unit squares and two line segments connecting two pairs of vertices are shown. If the area of
∆ABC is N, find 5N.

95. In rectangle ABCD, AB = 6, AD= 30, and G is the midpoint of AD . Segment AB is extended 2 units
beyond B to point E, and F is the intersection of ED and BC . If the area of BFDG is X, find [X].
96. Square EFGH has one vertex on each side of square ABCD. Point E is on AB with AE = 7 . EB. If the
m
ratio of the area of EFGH to the area of ABCD is , where m, n are co-prime, find m + n.
n
97. Rectangle ABCD has AB = 6 and BC = 3. Point M is chosen on side AB so that ∠AMD = ∠CMD.
What is the degree measure of ∠AMD?
98. Points A and C lie on a circle centered at O, each of BA and BC are tangent to the circle, and ∆ABC
is equilateral. The circle intersects BO at D. What is BD + BO?
99. A 9 × 9 × 9 cube is composed of twenty-seven 3 × 3 × 3 cubes. The big cube is 'tunneled' as follows:
First, the six 3 × 3 × 3 cubes which make up the center of each face as well as the center 3 × 3 × 3 cube
are removed as shown. Second, each of the twenty remaining 3 × 3 × 3 cubes is diminished in the same
way. That is, the center facial unit cubes as well as each center cube are removed. The approximate
square root of the surface area of the final figure is

100. A sector with acute central angle 60° is cut from a circle of radius 6. The radius of the circle
circumscribed about the sector is given by r. Find the value of 3 3 r.
101. If sin x + cos x = 1/5 and 0 ≤ x < π, then value of (–9 tan x) is
102. Suresh has 30 thin rods, one each of every integer length from 1 cm through 30 cm. She places the rods
with lengths 3 cm, 7 cm, and 15 cm on a table. She then wants to choose a fourth rod that she can put
with these three to form a quadrilateral with positive area. How many of the remaining rods can she
choose as the fourth rod?
103. Equiangular hexagon ABCDEF has side lengths AB =CD= EF = 1 and BC =DE= FA= r. The area of
∆ACE is 70% of the area of the hexagon. What is the sum of all possible values of r?
104. In the following diagram, ABCD is a square, and E is the center of the square ABCD. P is a point on a
semi-circle with diameter AB. Q is a point on a semi-circle with diameter AD. Moreover, Q, A and P
are collinear (that is, they are on the same line). Suppose QA = 14 cm, AP = 46 cm, and AE = x cm.
Find the value of x.

H.O. 92, Rajeev Gandhi Nagar, Kota (Raj.) Mob. 97831-97831, 70732-22177, Ph. 0744-2423333 www.nucleuseducation.in 14

14 14
PCCP

105. In the following diagram, ABCD is a square with PA =a, PB = 2a and PC = 3a.
Find the value of x where xº = 180° – ∠APB.

106. A solid cube of side length 1 is removed from each corner of a solid cube of side length 3. How many
edges does the remaining solid have?
107. ABC is a right-angled triangle with ∠BAC = 90°. A square is constructed on the side AB and BC as
shown. The area of the square ABDE is 8 cm2 and the area of the square BCFG is 26 cm2. Find the area
of triangle DBG in cm2.

108. One dimension of a cube is increased by 1, another is decreased by 1, and the third is left unchanged. The
volume of the new rectangular solid is 5 less than that of the cube. If the volume of the cube is V. Find 3 V .
109. A cylindrical tank with radius 4 feet and height 9 feet is lying on its side. The tank is filled with water
to a depth of 2 feet. If the volume of the water, in cubic feet is xπ + y 3 then find x + y?
110. A sphere is inscribed in a cube that has a surface area of 24 square meters. A second cube is then
inscribed within the sphere. What is the surface area in square meters of the inner cube?
111. Centers of adjacent faces of a unit cube are joined to form a regular octahedron. If the volume of this
octahedron is V, then find the value of 6V?
112. A wooden cube n units on a side is painted red on all six faces and then cut into n3 unit cubes. Exactly
one-fourth of the total number of faces of the unit cubes are red. What is n?
113. A white cylindrical silo has a diameter of 30 feet and a height of 80 feet. A red stripe with a horizontal
1
width of 3 feet is painted on the silo, as shown, making two complete revolutions around it. Find rd
3
of the area of the stripe in square feet.

H.O. 92, Rajeev Gandhi Nagar, Kota (Raj.) Mob. 97831-97831, 70732-22177, Ph. 0744-2423333 www.nucleuseducation.in 15

15 15
PCCP

114. An ice cream cone consists of a sphere of vanilla ice cream and a right circular cone that has the same
diameter as the sphere. If the ice cream melts, it will exactly fill the cone. Assume that the melted ice
cream occupies 75% of the volume of the frozen ice cream. If the ratio of the cones height to its radius
is given by a/b. Find sum of a & b.
11 5. The diagram show 28 lattice points, each one unit from its nearest neighbors. Segment AB meets
 9L2 
segment CD at E. Then the length of segment AE is given by L. Find the value of  .
 5 

116. All of the triangles in the diagram below are similar to iscoceles triangle ABC, in which AB = AC. Each of
the 7 smallest triangles has area 1, and ∆ABC has area 40. What is the area of trapezoid DBCE?

117. In the rectangular parallelopiped shown, AB = 3, BC = 1, and CG = 2. Point M is the midpoint of FG .


What is the volume of the rectangular pyramid with base BCHE and apex M?

118. If A is the area of the shaded region of the given 8 × 5 rectangle. Find the value of 4A.

H.O. 92, Rajeev Gandhi Nagar, Kota (Raj.) Mob. 97831-97831, 70732-22177, Ph. 0744-2423333 www.nucleuseducation.in 16

16 16
PCCP

119. When the centers of the faces of the right rectangular prism shown below are joined to create an
octahedron, What is the volume of the octahedron?

120. Equilateral ∆ABC has side length 1, and squares ABDE, BCHI, CAFG lie outside the triangle. If the
area of hexagon DEFGHI is A Then find the value of (A – 3 )3.

121. Six regular hexagons surround a regular hexagon of side length 1 as shown. What is the square of area
of ∆ABC?

122. Eight semicircles line the inside of a square with side length 2 as shown. If the radius of the circle
a −1
tangent to all of these semicircles is given by , where a and b are prime number. Find the
b
product of a and b.

123. Convex quadrilateral ABCD has AB = 9 and CD= 12. Diagonals AC and BD intersect at E, AC = 14,
and ∆AED and ∆BEC have equal areas. What is AE?
124. For some positive integers p, there is a quadrilateral ABCD with positive integer side lengths, perimeter
p, right angles at B and C, AB = 2, and CD = AD. How many different values of p < 2015 possible?

H.O. 92, Rajeev Gandhi Nagar, Kota (Raj.) Mob. 97831-97831, 70732-22177, Ph. 0744-2423333 www.nucleuseducation.in 17

17 17
PCCP
125. Consider the 12-sided polygon ABCDEFGHIJKL, as shown. Each of its sides has length 4, and each
two consecutive sides form a right angle. Suppose that AG and CH meet at M. What is the area of
quadrilateral ABCM to the nearest integer?

126. A paint brush is swept along both diagonals of a square to produce the symmetric painted area, as
shown. Half the area of the square is painted. If the ratio of the side length of the square to the brush
width is m 2 + n then Find the value of (m2n)2?

Answer Key
Que . 1 2 3 4 5 6 7 8 9 10 11 12 13 14
Ans. 1 1 3 1 4 3 3 2 2 1 2 1 1 1

12
15. 16. 54 17. 60 18. 25 19. 13 20. 40 21. 12
5
22. 16 23. 19 24. 01 25. 02 26. 25 27. 30 28. 31
29. 16 30. 21 31. 07 32. 03 33. 50 34. 16 35. 15
36. 11 37. 20 38. 31 39. 24 40. 42 41. 41 42. 28
43. 26 44. 24 45. 96 46. 23 47. 10 48. 03 49. 09
50. 04 51. 19 52. 98 53. 85 54. 86 55. 06 56. 01
57. 04 58. 08 59. 75 60. 05 61. 02 62. 05 63. 20
64. 25 65. 21 66. 24 67. 40 68. 30 69. 49 70. 81
71. 04 72. 04 73. 07 74. 01 75. 34 76. 62 77. 09
78. 22 79. 98 80. 45 81. 20 82. 50 83. 64 84. 81
85. 48 86. 31 87. 10 88. 72 89. 06 90. 05 91. 25
92. 49 93. 6 94. 01 95. 67 96. 57 97. 75 98. 03
99. 32 100. 18 101. 12 102. 17 103. 6 104. 34 105. 45
106. 84 107. 06 108. 05 109. 12 110. 08 111. 01 112. 04
113. 80 114. 04 115. 25 116. 24 117. 02 118. 26 119. 10
120. 27 121. 27 122. 10 123. 06 124. 31 125. 18 126. 64

H.O. 92, Rajeev Gandhi Nagar, Kota (Raj.) Mob. 97831-97831, 70732-22177, Ph. 0744-2423333 www.nucleuseducation.in 18

18 18
Rourkela Workshop
PRMO
Daily Practice Problems
Sub. : Mathematics DPP : 15
INEQUALITY
1. Which of the following is true?
x2 + 2 x2 + 2 x2 + 2 x2 + 2
(1) ≥5 (2) ≤2 (3) ≥2 (4) ≤5
x2 + 1 x2 + 1 x2 + 1 x2 + 1
2. If 0 < a < 1, then the value of the expression
 1+ a 1− a 1 1 
 + × 2 − 1 −  lies between
 1 + a − 1 − a 1− a2 −1+ a a a 
(1) –1 and +1 (2) –2 and + 2 (3) –3 and + 3 (4) –4 and + 4
14 11
3. Which is bigger 17 or 31
(1) 1714 > 3111 (2) 1714 < 31 11 (3) 1714 = 3111 (4) None of these
1 1
4. Without using tables, find that + is always greater than
log 2 π log 5 π
(1) 0 (2) 1 (3) 3/2 (4) 2
a b c
5. Let a, b, c be real numbers with 0 <a< 1, 0 < b < 1, 0 < c < 1 and a+ b + c = 2 then . . is
1− a 1− b 1− c
always greater than or equal to
(1) 2 (2) 4 (3) 6 (4) 8
1 1 1
6. If a, b, c are positive and a + b + c = 1. Then + + is greater than or equal to
a b c
(1) 6 (2) 9 (3) 12 (4) 8
7. If a, b, c, d are four non-negative real numbers and a + b + c + d = 1 then ab + bc + cd is less than equal to
(1) 1 (2) 2 (3) 3/2 (4) 1/4
8. For any three positive real numbers a, b and c, find the sign of a2 + b2 + c2 – ab – bc – ca
(1) is positive or equal to zero (2) is negative or equal to zero
(3) is neither positive or negative (4) cannot be found
9. For any real numbers x, y, z find minimum value of x4 + y4 + z4 – x2y2 – x2z2 – y2z2
(1) 0 (2) 1 (3) 2 (4) 3
a b c
10. For positive a, b, c, find minimum value of + +
b+c c+a a+b
1 3
(1) (2) 1 (3) (4) 2
2 2
1 1 1 1
11. If a, b, c, d are positive real numbers find minimum value of (a+ b + c + d)  + + + 
a b c d
(1) 0 (2) 4 (3) 8 (4) 16
12. It x, y, z are positive unequal reals x < y < z, then which of the following is true?
x 2 x 2 + y2 + z2 z2 x 2 x 2 + y2 + z2 x 2 + y2 + z2 z2
(1) > > (2) > OR <
z x+y+z x z x+y+z x+y+z x
x 2 x 2 + y2 + z2 z2 x 2 x 2 + y2 + z2 z2
(3) < > (4) < <
z x+y+z x z x+y+z x

H.O. 92, Rajeev Gandhi Nagar, Kota (Raj.) Mob. 97831-97831, 70732-22177, Ph. 0744-2423333 www.nucleuseducation.in 1

1 1
PCCP
13. n is a natural number greater than 1, and
n +1 n+4 n+7 n + 10 n + 13
A= + + + +
n n+3 n+6 n+9 n + 12
1 1 1 1 1
B= + + + + , then
n −1 n+2 n+5 n +8 n + 11
(1) A= B (2) A= 2B (3) A < B (4) A > B
14. The minimum value of the terms of the sequence
7 96 8 96 9 96 95 96
+ , + , + ……, + is
6 7 6 8 6 9 6 95
(1) 6 (2) 7 (3) 8 (4) 4
1
15. a and b are the roots of the quadratic equation x2 + λx – = 0 where x is the unknown and λ is a
2λ 2
real parameter. The minimum value of a4 + b4 is
2
(1) 2 2 (2) (3) 2 (4) 2 + 2
1+ 2
16. For positive unequal a and b, then which of the following is true?
(1) 3ab2 > a3 + 2b3 (2) 3ab2 < a3 + 2b3 (3) 3ab2 = a3 + 2b3 (4) None of these

17. If a and b are real positive distinct numbers. Then which one is true?
(1) a3b + ab3 > a4 + b4 (2) a3b + ab3 = a4 + b4 (3) a3b + ab3 < a4 + b4 (4) None of these
18. If a and b are positive real numbers such that ab = 1, then the least value of the expression (1 + a)(l + b) is
(1) 2 (2) 3 (3) 4 (4) 5
19. If a, b, c are the sides of a triangle, then
a b c
+ +
b+c−a c+a−b a +b−c
(1) ≤ 3 (2) ≥ 3 (3) ≥ 2 (4) ≤ 2
20. If a2 + b2 + c2 = 1, then for ab + bc + ca, which one is true?
1 1
(1) – ≤ ab + bc + ca ≤ 1 (2) ≤ ab + bc + ca ≤ 1
2 2
(3) –2 ≤ ab + bc + ca ≤ 1 (4) –2 ≤ ab + bc + ca ≤ –1
21. The function f(n) is defined for all positive integer n and take on non-negative integer values such that
f(2) = 0, f(3) > 0 and f(9999) = 3333. Also, for all m, n,
f(m + n) – f(m) – f(n) = 0 or 1.
Determine sum of all the digits of f(2005).
100 x 200
22. Let x and y be positive integers such that < < . What is the minimum value of x?
151 y 251
23. Let f(x) = ax2 + bx + c, where a, b and c are integers. Suppose that f(1) = 0, 50< f(7) < 60, 70 < f(8) <
80, and 5000k < f(100) < 5000 (k + 1) for some integer k. What is 3k2?
x
24. Given that x and y are positive integers such that 56 ≤ x + y ≤ 59 and 0.9 < < 0.91, find the value of
y
 x
y+ 4 .
 

ANSWER KEY
Que . 1 2 3 4 5 6 7 8 9 10 11 12 13 14 15 16 17 18 19 20
Ans. 3 2 1 4 4 2 4 1 1 3 4 4 3 4 4 2 3 3 2 1

21. 20 22. 03 23. 27 24. 38

H.O. 92, Rajeev Gandhi Nagar, Kota (Raj.) Mob. 97831-97831, 70732-22177, Ph. 0744-2423333 www.nucleuseducation.in 2

2 2

Potrebbero piacerti anche